Respiratory

¡Supera tus tareas y exámenes ahora con Quizwiz!

The nurse is assessing a 7-year-old boy with pharyngitis. The nurse would least likely expect to assess which of the following? A) Working hard to breathe B) Difficulty swallowing C) Rash on the abdomen D) Sore throat and headache

A) Working hard to breathe

When the nurse is reinforcing teaching with the caregiver of a 3-year-old child being discharged following a tonsillectomy, the caregiver states to the nurse, "I understand why there might be bleeding in the first 24 hours, but I do not understand why there might be bleeding in 1 week or so." What is the most appropriate explanation for the nurse to give this caregiver? A. "Bleeding can occur at this time because the clots dissolve and new tissue is not yet present." B. "We do not usually do this surgery until the child is older, so postoperative bleeding is a possible complication because of the child's age." C. "By next week the child will be eating regular foods again, and the salt content may cause bleeding." D. "The child will have forgotten about the surgery by that time and might start coughing, and the pressure of coughing can cause bleeding."

A. "Bleeding can occur at this time because the clots dissolve and new tissue is not yet present." Rationale: Hemorrhage is the most common complication of a tonsillectomy. Bleeding is most often a concern within the first 24 hours after surgery and up to the 10th postoperative day. Bleeding late postoperatively can occur when the clots dissolve and new tissue is not yet present. A tonsillectomy can be done at any age so stating that bleeding is a complication of age is incorrect. By 10 days postoperatively the child may still have a slight sore throat or have difficulty eating some solid foods so the child has not forgotten about the surgery. The pressure of coughing is most likely to cause bleeding early postoperatively. Salt will not cause bleeding and telling that to a parent is providing false information.

A nurse is caring for a client after a thoracentesis. Which sign, if noted in the client, should be reported to the physician immediately? A. "Client is becoming agitated and complains of pleuritic pain." B. "Client is drowsy and complains of headache." C. "Client has subcutaneous emphysema around needle insertion site." D. "Client has oxygen saturation of 93%."

A. "Client is becoming agitated and complains of pleuritic pain." Rationale: After a thoracentesis, the nurse monitors the client for pneumothorax or recurrence of pleural effusion. Signs and symptoms associated with pneumothorax depend on its size and cause. Pain is usually sudden and may be pleuritic. The client may have only minimal respiratory distress, with slight chest discomfort and tachypnea, and a small simple or uncomplicated pneumothorax. As the pneumothorax enlarges, the client may become anxious and develop dyspnea with increased use of the accessory muscles.

The nurse is teaching the parent of a child with cystic fibrosis about nutrition requirements for the child. What should be included in this teaching? Select all that apply. A. "Give your child high-calorie foods and snacks." B. "Feed your child foods that are high in protein." C. "Administer water soluble vitamins." D. "Give panreatic enzymes with meals." E. "Give your child foods high in fat."

A. "Give your child high-calorie foods and snacks." B. "Feed your child foods that are high in protein." D. "Give panreatic enzymes with meals." Rationale: Children with cystic fibrosis (CF) have trouble digesting and absorbing nutrients. They tend to be underweight. For optimal health, their diets should be high in calories and high in protein, with the supplementation of fat soluble vitamins and pancreatic enzymes. This diet helps with growth and the optimal nutrients. The fat soluble vitamins (vitamins A, D, E and K) are needed, because children with CF have trouble absorbing fat and need the vitamin supplementation to aid in fat absorption. Water soluble vitamins (the B vitamins and vitamin C) do not aid in fat absorption. The child should not have a high-fat diet, because the extra fat is difficult to digest and be absorbed. Pancreatic enzymes are necessary because they are missing due to the disease process. They are necessary to aid in digestion. They should be ingested with meals.

A child with asthma has been monitoring his peak expiratory flow rate (PEFR) and has been maintaining it within 90% of his personal best. Today, the child is experiencing symptoms and his PEFR is at 40% of his personal best. The child's mother calls the office and asks the nurse what she should do. What would the nurse instruct the mother to do first? A. "Have him use his short-acting bronchodilator right away." B. "You need to take him to the emergency department right away." C. "Continue to watch his PEFR readings and call back if they go below 40%." D. "Have him use his low-dose steroid inhaler now and again in 15 minutes."

A. "Have him use his short-acting bronchodilator right away." Rationale: The child's symptoms and drop in PEFR suggest a medical alert or "red" situation, indicating the need for the short-acting bronchodilator and then a trip to the office or emergency department. The child should use his short-acting bronchodilator first and then go to the physician's or nurse practitioner's office or emergency room. Waiting for a greater drop in his PEFR readings would be inappropriate because the child is experiencing an acute condition that warrants immediate attention. The child is experiencing an acute situation and requires immediate attention. A low-dose steroid inhaler would not be appropriate because it would not help his bronchospasm.

The nurse is obtaining a health history of a child suspected of tuberculosis. What question would the nurse ask first about the child's cough? A. "How long has your child had a cough?" B. "Does your child cough only at night?" C. "Does your child cough up anything when coughing?" D. "Has your child been around anyone who is coughing?"

A. "How long has your child had a cough?" Rationale: Tuberculosis is a highly contagious disease. Most children contract it from an infected immediate household member. When taking the health history, the nurse should ask about symptoms such as malaise, weight loss, anorexia, chest tightness, and a cough. The child's cough from tuberculosis is described as progressing slowly over several weeks and months rather than having an acute onset. Asking about the production from the cough is a way to determine if hemoptysis has occurred. Asking about being around anyone coughing is a way to determine if the child has been exposed to anyone with tuberculosis. Coughing only at night could be related to other respiratory disorders such as asthma.

The nurse is assessing a client who has a 35-pack-year history of cigarette smoking. In light of this known risk factor for lung cancer, which statement by the client should prompt the nurse to refer the client for further assessment? A. "Lately, I have this cough that just never seems to go away." B. "I find that I don't have nearly the stamina that I used to." C. "I seem to get nearly every cold and flu that goes around my workplace." D. "I never used to have any allergies, but now I think I'm developing allergies to dust and pet hair."

A. "Lately, I have this cough that just never seems to go away." Rationale: The most frequent symptom of lung cancer is cough or change in a chronic cough. People frequently ignore this symptom and attribute it to smoking or a respiratory infection. A new onset of allergies, frequent respiratory infections, and fatigue are not characteristic early signs of lung cancer.

A parent with a child who has cystic fibrosis asks the nurse how to determine if the child is receiving an adequate amount of pancreatic enzymes. How should the nurse respond? Select all that apply. A. "The dose is adequate when your child is only having 1 to 2 stools per day." B. "The dose is adequate when your child's weight is improving." C. "The dose prescribed is based on your child's pancreatic laboratory values so it should be correct." D. "When your child starts to eat more quantity of food you will need to adjust the amount of enzyme pills." E. "You will need to give your child less enzyme pills when high-fat foods are eaten."

A. "The dose is adequate when your child is only having 1 to 2 stools per day." B. "The dose is adequate when your child's weight is improving." D. "When your child starts to eat more quantity of food you will need to adjust the amount of enzyme pills." Rationale: Pancreatic enzymes are required for the child with cystic fibrosis (CF) to help absorb nutrients from the diet and to aid in digestion. They are given with each meal and snack the child eats. The number of capsules required at each dose depends upon the diagnosis of how the pancreas is functioning and the amount of food needing to be digested. The pancreatic laboratory values may detemine a baseline for the number of pills to start with, but the dosage is adjusted regularly. The dosage of pancreatic enzymes is adjusted until an adequate growth pattern is established and the child is having no more than 1 to 2 stools per day. The child should be given an increased number of enzyme pills when a meal with high-fat content is consumed, not fewer.

The school nurse is presenting a class on smoking cessation at the local high school. A participant in the class asks the nurse about the risk of lung cancer in those who smoke. What response related to risk for lung cancer in smokers is most accurate? A. "The younger you are when you start smoking, the higher your risk of lung cancer." B. "The risk for lung cancer never decreases once you have smoked, which is why smokers need annual chest x-rays." C. "The risk for lung cancer is determined mostly by what type of cigarettes you smoke." D. "The risk for lung cancer depends primarily on the other risk factors for cancer that you have."

A. "The younger you are when you start smoking, the higher your risk of lung cancer." Rationale: Risk is determined by the pack-year history (number of packs of cigarettes used each day, multiplied by the number of years smoked), the age of initiation of smoking, the depth of inhalation, and the tar and nicotine levels in the cigarettes smoked. The younger a person is when he or she starts smoking, the greater the risk of developing lung cancer. Risk declines after smoking cessation. The type of cigarettes is a significant variable, but this is not the most important factor.

A young child is prescribed pancreatic enzymes as part of the treatment plan for cystic fibrosis. The child has difficulty swallowing medications. After teaching the parents of a young child with cystic fibrosis about how to administer pancreatic enzymes, the parents demonstrate understanding by stating: A. "We can open the capsule and sprinkle it on his cereal." B. "We need to dissolve the capsule in water." C. "We should crush the capsule to make it smaller pieces." D. "We can puncture the capsule and pour the liquid on our child's tongue."

A. "We can open the capsule and sprinkle it on his cereal." Rationale: If the child has difficulty swallowing the pancreatic enzyme capsules, the parents can open the capsule and sprinkle the contents onto the child's cereal or applesauce. Dissolving the capsule in water or crushing it would be appropriate. The capsule does not contain liquid so there would not be any liquid to pour on the child's tongue.

The nurse is teaching home care to the parents of a 4-year-old client diagnosed with asthma. The nurse knows additional teaching is needed if the parents make which statement? A. "We will not enroll our child in preschool this year." B. "It is important for us to know what triggers an attack." C. "We feel confident we can administer medication with a nebulizer." D. "Emergency instructions and phone numbers are posted in our home."

A. "We will not enroll our child in preschool this year." Rationale: Enrolling in preschool is fine for children with asthma. There is the risk for increased exposure to illness; however, asthma should not limit the child from living a normal, active life. Should the child enroll, the nurse can assist in meeting the asthma education needs of the preschool staff through counseling the parents and providing access to accurate asthma education materials. The parents should know what triggers an asthma attack in their child, have emergency information and numbers posted in the home for emergency situations, and should know how to use a nebulizer as needed.

Acute respiratory failure (ARF) occurs when oxygen tension (PaO2) falls to less than __________ mm Hg (hypoxemia) and carbon dioxide tension (PaCO2) rises to greater than __________ mm Hg (hypercapnia). A. 60; 50 B. 60; 40 C. 75; 50 D. 75; 40

A. 60; 50 Rationale: Acute respiratory failure (ARF) is classified as hypoxemic (decrease in arterial oxygen tension [PaO2] to less than 60 mm Hg on room air) and hypercapnic (increase in arterial carbon dioxide tension [PaCO2] to greater than 50 mm Hg with an arterial pH of less than 7.35).

Which statement describes emphysema? A. A disease of the airways characterized by destruction of the walls of overdistended alveoli B. A disease that results in reversible airflow obstruction, a common clinical outcome C. Presence of cough and sputum production for at least a combined total of 2 to 3 months in each of two consecutive years D. Chronic dilatation of a bronchus or bronchi

A. A disease of the airways characterized by destruction of the walls of overdistended alveoli Rationale: Emphysema is a category of chronic obstructive pulmonary disease (COPD). In emphysema, impaired oxygen and carbon dioxide exchange results from destruction of the walls of overdistended alveoli. Emphysema is a pathologic term that describes an abnormal distention of the airspaces beyond the terminal bronchioles and destruction of the walls of alveoli; a chronic inflammatory response may induce disruption of the parenchymal tissues. Asthma has a clinical outcome of airflow obstruction. Bronchitis includes the presence of cough and sputum production for at least a combined total of 2 to 3 months in each of two consecutive years. Bronchiectasis is a condition of chronic dilatation of a bronchus or bronchi.

Which of the following is accurate regarding status asthmaticus? A. A severe asthma episode that is refractory to initial therapy B. Patients have a productive cough. C. Usually occurs with warning D. Usually does not progress to severe obstruction

A. A severe asthma episode that is refractory to initial therapy Rationale: Status asthmaticus is a severe asthma episode that is refractory to initial therapy. It is a medical emergency. Patients report rapid progressive chest tightness, wheezing, dry cough, and shortness of breath. It may occur with little or no warning.

A child is in the emergency department with an asthma exaccerbation. Upon asucultation the nurse is unable to hear air movement in the lungs. What action should the nurse take first? A. Administer a beta-2 adrenergic agonist B. Administer oxygen C. Start a peripheral IV D. Administer corticosteroids

A. Administer a beta-2 adrenergic agonist Rationale: When lungs sounds are unable to be heard in a child with asthma, the child is very ill. This means there is severe airway obstruction. The air movement is so severe wheezes cannot be heard. The priority treatment is to administer an inhaled short term bronchodilator (beta-2 adrenergic agonist). The child may require numerous inhalations until bronchodilation occurs and air can pass through the bronchi. Oxygen can be started but until the brochi are dilated no oxygen can get through to the lung fields. In IV would need to be started and IV steroids administered to reduce the inflammation, but the priority is bronchodilation.

While planning a client's care, the nurse identifies nursing actions to minimize the client's pleuritic pain. Which intervention should the nurse include in the plan of care? A. Administer an analgesic before coughing and deep breathing. B. Ambulate the client at least three times daily. C. Arrange for a soft-textured diet and increased fluid intake. D. Encourage the client to speak as little as possible.

A. Administer an analgesic before coughing and deep breathing. Rationale: The key characteristic of pleuritic pain is its relationship to respiratory movement. Taking a deep breath, coughing, or sneezing worsens the pain. Because deep breathing and coughing prevent atelectasis, the client should be given an analgesic prior to performing these respiratory exercises. A soft diet is not necessarily indicated, and there is no need for the client to avoid speaking. Ambulation has multiple benefits, but pain management is not among them.

The nurse getting an end-of-shift report on a child with status asthmatics should question which intervention? A. Administer oxygen by nasal cannula to keep oxygen saturation at 100% B. Assess IV maintenance fluids and site every hour C. Notify physician for signs of increasing respiratory distress D. Organize care to allow for uninterrupted rest periods

A. Administer oxygen by nasal cannula to keep oxygen saturation at 100% Rationale: Supplemental oxygen should not be administered to maintain oxygen saturation at 100%. Keeping the saturation around 95% is adequate. Administration of too much oxygen to a child may lead to a respiratory depression by decreasing stimulus to breath, leading to CO2 retention.

A 3-year-old child with asthma and a respiratory tract infection is prescribed an antibiotic and a bronchodilator. The nurse notes the following during assessment: oral temperature 100.2°F (37.9°C), respirations 52 breaths/minute, heart rate 90 beats/minute, O2 saturation 95% on room air. Which action will the nurse take first? A. Administer the bronchodilator via a nebulizer. B. Give the antibiotic as prescribed. C. Apply oxygen at 2 liters via a nasal cannula. D. Apply a cardiac monitor to the child.

A. Administer the bronchodilator via a nebulizer. Rationale: The nurse would first administer the bronchodilator to open the child's airway and facilitate breathing. Once the airway was open, the nurse could administer oxygen, if indicated. At this time, the child's saturation level is normal but it should be monitored. The nurse would then administer the antibiotic medication. The heart rate is within normal range for a child of this age (65 to 110 beats/minute); therefore, a cardiac monitor is not needed at this time.

A 4-year-old child has been admitted to the hospital with a diagnosis of pneumococcal pneumonia. The parents are extremely distraught over the child's condition and the fact that the child has not wanted to eat anything for the past 2 days. Which nursing approach would be most important to take to help alleviate the high anxiety level of the parents? A. Allow the parents to remain with the child as much as possible. B. Encourage the parents to return home and get some rest. C. Tell the parents that their child is receiving the best care possible. D. Avoid telling the parents unnecessary facts regarding the child's prognosis.

A. Allow the parents to remain with the child as much as possible. Rationale: Pneumonia may be caused by many reasons: bacteria, viruses, fungus, and aspiration. If the child has mild symptoms (no respiratory distress) he or she may be treated at home. Hospitalization is required if the child has oxygen requirements, shows signs of respiratory distress, has poor oral intake, and has lethargy. Oxygen supplementation, IV fluids, and antibiotics will be necessary. It is very frightening for the parents to see their child so ill, and it is very frightening for the child to be so sick and be in a strange environment. The parents should be allowed to remain with their child at all times and their concerns should be addressed. The nurse should explain that not eating is part of the illness, but the child is being hydrated with IV fluids and will start eating as the illness improves. Telling the parents the child is receiving the best care possible does not address their concern of not eating. Parents should be educated on all aspects of the child's condition and prognosis.

A nurse is caring for a client with status asthmaticus. Which medication should the nurse prepare to administer? A. An inhaled beta2-adrenergic agonist B. An inhaled corticosteroid C. An I.V. beta2-adrenergic agonist D. An oral corticosteroid

A. An inhaled beta2-adrenergic agonist Rationale: An inhaled beta2-adrenergic agonist helps promote bronchodilation, which improves oxygenation. Although an I.V. beta2-adrenergic agonist can be used, the client needs be monitored because of the drug's greater systemic effects. The I.V. form is typically used when the inhaled beta2-adrenergic agonist doesn't work. A corticosteroid is slow acting, so its use won't reduce hypoxia in the acute phase.

A client arrives in the emergency room with emphysema and has developed an exacerbation of COPD with respiratory acidosis from airway obstruction. What is the highest priority for the nurse? A. Apply supplemental oxygen as ordered. B. Assess vital signs every 2 hours, including O2 saturations and ABG results. C. Educate the client about the importance of pursed lip breathing. D. Refer the client to respiratory therapy if breathing becomes labored.

A. Apply supplemental oxygen as ordered. Rationale: When the client arrives in an ED, the first line of treatment is supplemental oxygen therapy and rapid assessment. Oxygen will correct the hypoxemia. Careful observation of the liter flow or the percentage administered and its effect on the patient is important. These clients generally require low-flow oxygen rates of 1-2 L/min. Monitor and titrate to achieve desired PaO2. Periodic arterial blood gases and pulse oximetry help evaluate the adequacy of oxygenation.

The nurse is caring for a 5-year-old client and notes respiratory rate of 45 breaths per minute, blood pressure 100/70 mm Hg, heart rate 115, temperature 101°F (38.3°C), and oxygen saturation 86%. Which diagnostic test is priority for the nurse to complete? A. Arterial blood gas (ABG) B. Complete blood count (CBC) C. Electroencephalogram (EEG) D. Pulmonary function test

A. Arterial blood gas (ABG) Rationale: The most useful diagnostic test in respiratory distress is an ABG. Knowing normal blood gas values for children is very important for evaluation and proper treatment. A CBC is a blood test used to test for disorders including anemia, infection, and leukemia. An EEG is a test used to find problems related to electrical activity of the brain. A pulmonary function test is performed to evaluate the respiratory system. Based on the findings, the child is experiencing respiratory distress and has an elevated temperature. Airway and breathing are priority over an elevated temperature. The child's blood pressure is within normal range for this age.

Which of the following is a potential complication of a low pressure in the endotracheal cuff? A. Aspiration pneumonia B. Tracheal bleeding C. Tracheal ischemia D. Pressure necrosis

A. Aspiration pneumonia Rationale: Low pressure in the cuff can increase the risk for aspiration pneumonia. High cuff pressure can cause tracheal bleeding, ischemia, and pressure necrosis.

A child has been prescribed a nasal cannula for oxygen delivery. What should the nurse do before applying the cannula? A. Assess patency of the nares B. Test the oxygen saturation C. Add humidification to the delivery device D. Assess the lung sounds

A. Assess patency of the nares Rationale: A nasal cannula is a good delivery device for children, because it allows them to eat and talk unobstructed. Because the device is designed for flow through the nares, the patency of the nares should be assessed prior to using the cannula. If the nares are blocked from secretions, suctioning may be required. If there is a defect in the upper airway causing blockage, the nasal cannula may not be an appropriate oxygen delivery device. The oxygen saturation should have been measured and used as a guide for the prescription of oxygen therapy. Adding humidification is a way to keep the upper airways from becoming too dry, but oxygen can be started before humidity is added. Anytime a child is sick enough to require oxygen all respiratory assessments, including lung sounds, should be done. It does not matter, however, what the lung sounds are if the child is in enough distress to require oxygen. The lung sounds can be assessed after oxygen is started.

The nurse is teaching the parents about medications for their 9-year-old boy who has a respiratory disorder. The nurse would be alert for an increased need for medications if the child was exposed to second-hand smoke and has which condition? A. Asthma B. Common cold C. Pneumonia D. Allergic rhinitis (hay fever)

A. Asthma Rationale: In general, it is important for any child with a respiratory illness to avoid second-hand smoke. However, exposure to second-hand smoke increases the need for medications in children with asthma and increases the frequency of asthma exacerbations. In general, it is important for any child with a respiratory illness to avoid second-hand smoke. However, the presence of smoke does not increase the medication needs for children with a cold. In general, it is important for any child with a respiratory illness to avoid second-hand smoke. However, the presence of smoke does not increase the medication needs for children with pneumonia. In general, it is important for any child with a respiratory illness to avoid second-hand smoke. However, the presence of smoke does not increase the medication needs for children with allergic rhinitis (hay fever).

A child has a chronic, nonproductive cough and diffuse wheezing during the expiratory phase of respiration. This suggests: A. Asthma B. Pneumonia C. Bronchiolitis D. Foreign body in trachea

A. Asthma Rational: Children with asthma usually have these chronic symptoms.

The nurse should be alert for a complication of bronchiectasis that results from a combination of retained secretions and obstruction and that leads to the collapse of alveoli. What complication should the nurse monitor for? A. Atelectasis B. Emphysema C. Pleurisy D. Pneumonia

A. Atelectasis Rationale: In bronchiectasis, the retention of secretions and subsequent obstruction ultimately cause the alveoli distal to the obstruction to collapse (atelectasis).

A new ICU nurse is observed by her preceptor entering a patient's room to suction the tracheostomy after performing the task 15 minutes before. What should the preceptor educate the new nurse to do to ensure that the patient needs to be suctioned? A. Auscultate the lung for adventitious sounds. B. Have the patient inform the nurse of the need to be suctioned. C. Assess the CO2 level to determine if the patient requires suctioning. D. Have the patient cough.

A. Auscultate the lung for adventitious sounds. Rationale: When a tracheostomy or endotracheal tube is in place, it is usually necessary to suction the patient's secretions because of the decreased effectiveness of the cough mechanism. Tracheal suctioning is performed when adventitious breath sounds are detected or whenever secretions are obviously present. Unnecessary suctioning can initiate bronchospasm and cause mechanical trauma to the tracheal mucosa.

The nurse is caring for an infant whose oxygen saturation levels frequently drop below 90%. Which data is most important to relate to the health care provider? A. Blood gases B. Vital signs C. Respiratory depth and pattern D. Breath sounds

A. Blood gases Rationale: Infants may respond to low blood oxygen levels with increased respirations followed by a period of apnea. Conditions such as bronchopulmonary dysplasia (chronic lung disease), pneumonia, and bronchiolitis can put infants at risk. The health care provider needs to be kept updated on blood oxygen levels. Vital signs, respiratory depth, and pattern, and breath sounds are basic nursing assessments that provide helpful data on the respiratory system, but these data are not as important as the laboratory results.

A school-age child has an upper respiratory tract infection for several days and then began having a persistent dry, hacking cough that was worse at night. The cough has become productive in the past 24-hours. This is most suggestive of: A. Bronchitis B. Bronchiolitis C. Viral-induced asthma D. Acute spasmodic laryngitis

A. Bronchitis Rationale: Bronchitis is characterized by these symptoms and occurs in children older than 6 years.

A nurse is working with a 10-year-old client who is undergoing a diagnostic workup for suspected asthma. Which signs and symptoms are consistent with a diagnosis of asthma? Select all that apply. A. Chest tightness B. Crackles C. Bradypnea D. Wheezing E. Cough

A. Chest tightness D. Wheezing E. Cough Rationale: Asthma is a chronic inflammatory disease of the airways that causes airway hyperresponsiveness, mucosal edema, and mucus production. This inflammation ultimately leads to recurrent episodes of asthma symptoms: cough, chest tightness, wheezing, and dyspnea. Crackles and bradypnea are not typical symptoms of asthma.

The nurse is caring for a 14-month-old boy with cystic fibrosis. Which sign of ineffective family coping requires urgent and immediate intervention? A. Compliance with therapy is diminished. B. The family becomes overvigilant. C. The child feels fearful and isolated. D. Siblings are jealous and worried.

A. Compliance with therapy is diminished. Rationale: Until the family adjusts to the demands of the disease, they can become overwhelmed and exhausted, leading to noncompliance, resulting in worsening of symptoms. Typical challenges to the family are becoming overvigilant, the child feeling fearful and isolated, and the siblings being jealous or worried, but these are not a priority over the noncompliance.

Upon assessment, the nurse suspects that a client with COPD may have bronchospasm. What manifestations validate the nurse's concern? Select all that apply. A. Compromised gas exchange B. Decreased airflow C. Wheezes D. Jugular vein distention E. Ascites

A. Compromised gas exchange B. Decreased airflow C. Wheezes Rationale: Bronchospasm, which occurs in many pulmonary diseases, reduces the caliber of the small bronchi and may cause dyspnea, static secretions, and infection. Bronchospasm can sometimes be detected on auscultation with a stethoscope when wheezing or diminished breath sounds are heard. Increased mucus production, along with decreased mucociliary action, contributes to further reduction in the caliber of the bronchi and results in decreased airflow and decreased gas exchange. This is further aggravated by the loss of lung elasticity that occurs with COPD.

What is the most common debilitating disease of childhood among those of European descent? A. Cystic fibrosis B. Asthma C. Pneumonia D. BPD

A. Cystic fibrosis Rationale: Cystic fibrosis is the most common debilitating disease of childhood among those of European descent. Medical advances in recent years have greatly increased the length and quality of life for affected children, with median age for survival being the late 30s.

An emergency room nurse is assessing a client who is complaining of dyspnea. Which sign would indicate the presence of a pleural effusion? A. Decreased chest wall excursion upon palpation B. Wheezing upon auscultation C. Resonance upon percussion D. Mottled skin seen during inspection

A. Decreased chest wall excursion upon palpation Rationale: Symptoms of pleural effusion are shortness of breath, pain, assumption of a position that decreases pain, absent breath sounds, decreased fremitus, a dull, flat sound upon percussion, and decreased chest wall excursion. The nurse may also hear a friction rub. Chest radiography and computed tomography show fluid in the involved area.

The nurse is caring for a client who is scheduled for a lobectomy for lung cancer. While assisting with a subclavian vein central line insertion, the nurse notes that the client's oxygen saturation is rapidly dropping. The client reports shortness of breath and becomes tachypneic. The nurse suspects a pneumothorax has developed. Which further assessment finding would support the presence of a pneumothorax? A. Diminished or absent breath sounds on the affected side B. Paradoxical chest wall movement with respirations C. Sudden loss of consciousness D. Muffled heart sounds

A. Diminished or absent breath sounds on the affected side Rationale: In the case of a simple pneumothorax, auscultating the breath sounds will reveal absent or diminished breath sounds on the affected side. Paradoxical chest wall movements occur in flail chest conditions. Sudden loss of consciousness does not typically occur. Muffled or distant heart sounds occur in pericardial tamponade.

What is the reason for chest tubes after thoracic surgery? A. Draining secretions, air, and blood from the thoracic cavity is necessary. B. Chest tubes allow air into the pleural space. C. Chest tubes indicate when the lungs have re-expanded by ceasing to bubble. D. Draining secretions and blood while allowing air to remain in the thoracic cavity is necessary.

A. Draining secretions, air, and blood from the thoracic cavity is necessary. Rationale: After thoracic surgery, draining secretions, air, and blood from the thoracic cavity is necessary to allow the lungs to expand.

The perioperative nurse is writing a care plan for a client who has returned from surgery 2 hours ago. Which measure should the nurse implement to most decrease the client's risk of developing pulmonary emboli (PE)? A. Early ambulation B. Increased dietary intake of protein C. Maintaining the client in a supine position D. Administering aspirin with warfarin

A. Early ambulation Rationale: For clients at risk for PE, the most effective approach for prevention is to prevent deep vein thrombosis. Active leg exercises to avoid venous stasis, early ambulation, and use of elastic compression stockings are general preventive measures. The client does not require increased dietary intake of protein directly related to prevention of PE, although it will assist in wound healing during the postoperative period. The client should not be maintained in one position, but frequently repositioned unless contraindicated by the surgical procedure. Aspirin should never be given with warfarin because it will increase the client's risk for bleeding.

A child is hospitalized with pneumonia. The nurse assesses an increase in the work of breathing and in the respiratory rate. What intervention should the nurse do first to help this child? A. Elevate the head of the bed B. Administer oxygen C. Notify the health care provider D. Obtain oxygen saturation levels

A. Elevate the head of the bed Rationale: The child who is experiencing increased work of breathing should be placed in a position to better open the airway and provide more room for lung expansion. Generally this is acomplished by elevating the head of the bed. If this does not improve the work of breathing, then administering oxygen should be done. The oxygen saturation should be measured because it will provide information as to the severity of the respiratory problem, but this measurement will not directly help the child. The health care provider should be notified if the child continues to deteriorate.

A nurse is evaluating the diagnostic study data of a client with suspected cystic fibrosis (CF). Which of the following test results is associated with a diagnosis of cystic fibrosis? A. Elevated sweat chloride concentration B. Presence of protein in the urine C. Positive phenylketonuria D. Decreased tidal volume

A. Elevated sweat chloride concentration Rationale: Gene mutations affect transport of chloride ions, leading to CF, which is characterized by thick, viscous secretions in the lungs, pancreas, liver, intestine, and reproductive tract as well as increased salt content in sweat gland secretions. Proteinuria, positive phenylketonuria, and decreased tidal volume are not diagnostic for CF.

A client is recovering from thoracic surgery needed to perform a right lower lobectomy. Which of the following is the most likely postoperative nursing intervention? A. Encourage coughing to mobilize secretions. B. Restrict intravenous fluids for at least 24 hours. C. Make sure that a thoracotomy tube is linked to open chest drainage. D. Assist with positioning the client on the right side.

A. Encourage coughing to mobilize secretions. Rationale: The client is encouraged to cough frequently to mobilize secretions. The client will be placed in the semi-Fowler's position. The chest tube is always attached to closed, sealed drainage to re-expand lung tissue and prevent pneumothorax. Restricting IV fluids in a client who is NPO while recovering from surgery would lead to dehydration.

The nurse is planning the care for a client at risk of developing pulmonary embolism. What nursing interventions should be included in the care plan? Select all that apply. A. Encouraging a liberal fluid intake B. Instructing the client to move the legs in a "pumping" exercise C. Instructing the client to dangle the legs over the side of the bed for 30 minutes, four times a day D. Using elastic stockings, especially when decreased mobility would promote venous stasis E. Applying a sequential compression device

A. Encouraging a liberal fluid intake B. Instructing the client to move the legs in a "pumping" exercise D. Using elastic stockings, especially when decreased mobility would promote venous stasis E. Applying a sequential compression device Rationale: The use of anti-embolism stockings or intermittent pneumatic leg compression devices reduces venous stasis. These measures compress the superficial veins and increase the velocity of blood in the deep veins by redirecting the blood through the deep veins. Having the client move the legs in a "pumping" exercise helps increase venous flow. Legs should not be dangled or feet placed in a dependent position while the client sits on the edge of the bed; instead, feet should rest on the floor or on a chair.

A nurse is caring for a client who has a tracheostomy and temperature of 103° F (39.4° C). Which intervention will most likely lower the client's arterial blood oxygen saturation? A. Endotracheal suctioning B. Encouragement of coughing C. Use of a cooling blanket D. Incentive spirometry

A. Endotracheal suctioning Rationale: Endotracheal suctioning removes secretions as well as gases from the airway and lowers the arterial oxygen saturation (SaO2) level. Coughing and using an incentive spirometer improve oxygenation and should raise or maintain oxygen saturation. Because of superficial vasoconstriction, using a cooling blanket can lower peripheral oxygen saturation readings, but SaO2 levels wouldn't be affected.

Which of the following is a common irritant that acts as a trigger of asthma? A. Esophageal reflux B. Peanuts C. Aspirin sensitivity D. Molds

A. Esophageal reflux Rationale: Esophageal reflux, viral respiratory infections, cigarette smoke, and exercise are all irritants that can trigger asthma. Peanuts, aspirin sensitivity, and molds are antigens

The nurse has instructed the client to use a peak flow meter. The nurse evaluates client learning as satisfactory when the client A. Exhales hard and fast with a single blow B. Inhales deeply and holds the breath C. Records in a diary the number achieved after one breath D. Sits in a straight-back chair and leans forward

A. Exhales hard and fast with a single blow Rationale: To use a peak flow meter, the client stands. Then the client takes a deep breath and exhales hard and fast with a single blow. The client repeats this twice and records a "personal best" in an asthma diary.

The occupational nurse is completing routine assessments on the employees at a company. What might be revealed by a chest radiograph for a client with occupational lung diseases? A. Fibrotic changes in lungs B. Hemorrhage C. Lung contusion D. Damage to surrounding tissues

A. Fibrotic changes in lungs Rationale: For a client with occupational lung diseases, a chest radiograph may reveal fibrotic changes in the lungs. Hemorrhage, lung contusion, and damage to surrounding tissues are possibly caused by trauma due to chest injuries.

A victim of a motor vehicle accident has been brought to the emergency room. The patient is exhibiting paradoxical chest expansion and respiratory distress. Which of the following chest disorders should be suspected? A. Flail chest B. Cardiac tamponade C. Pulmonary contusion D. Simple pneumothorax

A. Flail chest Rationale: When a flail chest exists, during inspiration, as the chest expands, the detached part of the rib segment (flail segment) moves in a paradoxical manner in that it is pulled inward during inspiration, reducing the amount of air that can be drawn into the lungs. On expiration, because the intrathoracic pressure exceed atmospheric pressure, the flail segment bulges outward, impairing the patient's ability to exhale. Cardiac tamponade is compression of the heart resulting from fluid or blood within the pericardial sac. A pulmonary contusion is damage to the lung tissues resulting in hemorrhage and localized edema. A simple pneumothorax occurs when air enters the pleural space through the rupture of a bleb or a bronchopleural fistula.

The nurse should assess a child who has a tonsillectomy for: A. Frequent swallowing B. Inspiratory stridor C. Rhonchi D. Elevated WBC count

A. Frequent swallowing Rationale: Frequent swallowing is indicative of postoperative bleeding.

A nurse is caring for a client who has been admitted with an exacerbation of chronic bronchiectasis. The nurse should expect to assess the client for which clinical manifestation? A. Hemoptysis B. Pain on inspiration C. Pigeon chest D. Dry cough

A. Hemoptysis Rationale: Clinical manifestations of bronchiectasis include hemoptysis, chronic cough, copious purulent sputum, and clubbing of the fingers. Because of the copious production of sputum, the cough is rarely dry. A pigeon chest or pectus carinatum is a deformity of the chest wall, with children and adolescents being typically affected. Pigeon chest is not associated with this disease. Pain on inspiration is usually associated with respiratory conditions such as pleurisy, pneumonia, or pneumothorax.

A pediatric nurse practitioner is caring for a 2-year-old client who has just been diagnosed with asthma. The nurse has provided the parents with information that includes potential causative agents for an asthmatic reaction. Which potential causative agent that may trigger an attack should the nurse describe? A. Household pets B. Inadequate sleep C. Psychosocial stress D. Bacteria

A. Household pets Rationale: Common causative agents that may trigger an asthma attack are as follows: dust, dust mites, pets, soap, certain foods, molds, and pollens. Although some research links inadequate sleep to making asthma worse, it is not a common trigger for attacks. Psychosocial stress is anything that translates to a perceived threat to social status. Stress is listed as a trigger for asthma, but this type of stress is unlikely in a 2-year-old client. A viral, not bacterial, component is linked to asthma triggers.

The nurse is providing care for a client who has recently been diagnosed with chronic obstructive pulmonary disease. When educating the client about exacerbations, the nurse should prioritize which topic? A. Identifying specific causes of exacerbations B. Prompt administration of corticosteroids during exacerbations C. The importance of prone positioning during exacerbations D. The relationship between activity level and exacerbations

A. Identifying specific causes of exacerbations Rationale: Prevention is key in the management of exacerbations, and it is important for the client to identify which factors cause exacerbations. Corticosteroids are not normally used as a "rescue" medication, and prone positioning does not enhance oxygenation. Activity in the morning may need to be delayed for an hour or two for bronchial secretions that have collected overnight in the lungs to clear. Therefore, the right amount of activity, at the right time, can impact exacerbations, but prevention is the priority.

Asthma is cause by which type of response? A. IgE-mediated B. IgA-mediated C. IgD-mediated D. IgM-mediated

A. IgE-mediated Rationale: Atopy, the genetic predisposition for the development of an IgE-mediated response to allergens, is the most common identifiable predisposing factor for asthma. Chronic exposure to airway allergens may sensitize IgE antibodies and the cells of the airway.

The nurse is caring for a client at risk for atelectasis. The nurse implements a first-line measure to prevent atelectasis development in the client. What is an example of a first-line measure to minimize atelectasis? A. Incentive spirometry B. Intermittent positive-pressure breathing (IPPB) C. Positive end-expiratory pressure (PEEP) D. Bronchoscopy

A. Incentive spirometry Rationale: Strategies to prevent atelectasis, which include frequent turning, early ambulation, lung-volume expansion maneuvers (deep breathing exercises, incentive spirometry), and coughing, serve as the first-line measures to minimize or treat atelectasis by improving ventilation. In clients who do not respond to first-line measures or who cannot perform deep-breathing exercises, other treatments such as positive end-expiratory pressure (PEEP), continuous or intermittent positive-pressure breathing (IPPB), or bronchoscopy may be used.

A critical-care nurse is caring for a client diagnosed with pneumonia as a surgical complication. The nurse's assessment reveals that the client has an increased work of breathing due to copious tracheobronchial secretions. What should the nurse encourage the client to do? A. Increase oral fluids unless contraindicated. B. Call the nurse for oral suctioning, as needed. C. Lie in a low Fowler or supine position. D. Increase activity.

A. Increase oral fluids unless contraindicated. Rationale: The nurse should encourage hydration because adequate hydration thins and loosens pulmonary secretions. Oral suctioning is not sufficiently deep to remove tracheobronchial secretions. The client should have the head of the bed raised, and rest should be promoted to avoid exacerbation of symptoms.

The nurse is administering 2 puffs of an albuterol sulfate inhaler to a 4-year-old. Which side effect would the nurse instruct the parent to most likely expect? A. Increased heart rate and restlessness B. Increased mucus expectoration C. Increased nonproductive cough D. Drowsiness causing a nap

A. Increased heart rate and restlessness Rationale: The nurse instructs the parents on the side effects of the bronchodilator, albuterol sulfate. The side effect of this medication is restlessness, anxiety, fear, palpitations, and tremors. It is important for the parents to realize this so they understand the actions of the 4-year-old. Once the bronchial tree is open, cough is decreased but mucus expectoration could increase. The medication does not cause drowsiness.

A nurse has been asked to give a workshop on chronic obstructive pulmonary disease for a local community group. The nurse emphasizes the importance of smoking cessation because smoking has which pathophysiologic effect? A. Increases the amount of mucus produced B. Destabilizes hemoglobin C. Shrinks the alveoli in the lungs D. Collapses the alveoli in the lungs

A. Increases the amount of mucus produced Rationale: Smoking irritates the goblet cells and mucous glands, causing an increased accumulation of mucus, which, in turn, produces more irritation, infection, and damage to the lung. Smoking is not known to destabilize hemoglobin, shrink the alveoli in the lungs, or collapse the alveoli in the lungs.

A nursing student is taking a pathophysiology examination. Which of the following factors would the student correctly identify as contributing to the underlying pathophysiology of chronic obstructive pulmonary disease (COPD)? Choose all that apply. A. Inflamed airways that obstruct airflow B. Mucus secretions that block airways C. Overinflated alveoli that impair gas exchange D. Dry airways that obstruct airflow E. Decreased numbers of goblet cells

A. Inflamed airways that obstruct airflow B. Mucus secretions that block airways C. Overinflated alveoli that impair gas exchange Rationale: Because of chronic inflammation and the body's attempts to repair it, changes and narrowing occur in the airways. In the proximal airways, changes include increased numbers of goblet cells and enlarged submucosal glands, both of which lead to hypersecretion of mucus. In the peripheral airways, inflammation causes thickening of the airway wall, peribronchial fibrosis, exudate in the airway, and overall airway narrowing.

Which of the following is the key underlying feature of asthma? A. Inflammation B. Shortness of breath C. Productive cough D. Chest tightness

A. Inflammation Rationale: Inflammation is the key underlying feature and leads to recurrent episodes of asthma symptoms: cough, chest tightness, wheeze, and dyspnea.

An x-ray of a trauma client reveals rib fractures, and the client is diagnosed with a small flail chest injury. Which intervention should the nurse include in the client's plan of care? A. Initiate chest physiotherapy. B. Immobilize the ribs with an abdominal binder. C. Prepare the client for surgery. D. Immediately sedate and intubate the client.

A. Initiate chest physiotherapy. Rationale: As with rib fracture, treatment of flail chest is usually supportive. Management includes chest physiotherapy and controlling pain. Intubation is required only for severe flail chest injuries, not small flail chest injuries, and surgery is required only in rare circumstances to stabilize the flail segment. Immobilization of the ribs with an abdominal binder is not necessary for a small flail chest injury.

A client who underwent surgery 12 hours ago has difficulty breathing. He has petechiae over his chest and complains of acute chest pain. What action should the nurse take first? A. Initiate oxygen therapy. B. Administer a heparin bolus and begin an infusion at 500 units/hour. C. Administer analgesics as ordered. D. Perform nasopharyngeal suctioning.

A. Initiate oxygen therapy. Rationale: The client's signs and symptoms suggest pulmonary embolism. Therefore, maintaining respiratory function takes priority. The nurse should first initiate oxygen therapy and then notify the physician immediately. The physician will most likely order an anticoagulant such as heparin or an antithrombolytic to dissolve the thrombus. Analgesics can be administered to decrease pain and anxiety but administering oxygen takes priority. Suctioning typically isn't necessary with pulmonary embolism.

The nurse is assisting a client with postural drainage. Which of the following demonstrates correct implementation of this technique? A. Instruct the client to remain in each position of the postural drainage sequence for 10 to 15 minutes. B. Use aerosol sprays to deodorize the client's environment after postural drainage. C. Perform this measure with the client once a day. D. Administer bronchodilators and mucolytic agents following the sequence.

A. Instruct the client to remain in each position of the postural drainage sequence for 10 to 15 minutes. Rationale: Postural drainage is usually performed two to four times daily, before meals (to prevent nausea, vomiting, and aspiration) and at bedtime. Prescribed bronchodilators, water, or saline may be nebulized and inhaled before postural drainage to dilate the bronchioles, reduce bronchospasm, decrease the thickness of mucus and sputum, and combat edema of the bronchial walls. The nurse instructs the client to remain in each position for 10 to 15 minutes and to breathe in slowly through the nose and out slowly through pursed lips to help keep the airways open so that secretions can drain while in each position. If the sputum is foul-smelling, it is important to perform postural drainage in a room away from other patients or family members. (Deodorizers may be used to counteract the odor. Because aerosol sprays can cause bronchospasm and irritation, they should be used sparingly and with caution.)

A client diagnosed with acute respiratory distress syndrome (ARDS) is restless and has a low oxygen saturation level. If the client's condition does not improve and the oxygen saturation level continues to decrease, what procedure will the nurse expect to assist with in order to help the client breathe more easily? A. Intubate the client and control breathing with mechanical ventilation B. Increase oxygen administration C. Administer a large dose of furosemide (Lasix) IVP stat D. Schedule the client for pulmonary surgery

A. Intubate the client and control breathing with mechanical ventilation Rationale: A client with ARDS may need mechanical ventilation to assist with breathing while the underlying cause of the pulmonary edema is corrected. The other options are not appropriate.

The nurse is educating the parents of a 7-year-old boy with asthma about the medications that have been prescribed. Which drug would the nurse identify as an adjunct to a β2-adrenergic agonist for treatment of bronchospasm? A. Ipratropium B. Montelukast C. Cromolyn D. Theophylline

A. Ipratropium Rationale: Ipratropium is an anticholinergic administered via inhalation to produce bronchodilation without systemic effects. It is generally used as an adjunct to a β2-adrenergic agonist. Montelukast decreases the inflammatory response by antagonizing the effects of leukotrienes. Cromolyn prevents release of histamine from sensitized mast cells. Theophylline provides for continuous airway relaxation.

What explanation should the nurse give the parent of a child with asthma about using a peak flow meter? A. It is used to monitor the childs breathing capacity B. It measures the childs lung volume C. It will help the medication reach the childs airway D. It measures the amount of air the child breaths in

A. It is used to monitor the childs breathing capacity Rationale: The peak flow meter is a device used to monitor breathing capacity in the child with asthma.

A client has a history of chronic obstructive pulmonary disease (COPD). Following a coughing episode, the client reports sudden and unrelieved shortness of breath. Which of the following is the most important for the nurse to assess? A. Lung sounds B. Skin color C. Heart rate D. Respiratory rate

A. Lung sounds Rationale: A client with COPD is at risk for developing pneumothorax. The description given is consistent with possible pneumothorax. Though the nurse will assess all the data, auscultating the lung sounds will provide the nurse with the information if the client has a pneumothorax.

An infant with a high respiratory rate is NPO and is receiving IV fluids. What assessment(s) will the nurse make to assure this infant is hydrated? Select all that apply. A. Measure skin turgor B. Palpate anterior fontanel C. Determine urine output D. Review electrolyte laboratory results E. Assess the lung sounds

A. Measure skin turgor B. Palpate anterior fontanel C. Determine urine output Rationale: IV fluids are necessary many times for infants and children who are experiencing high respiratory rates. The high respiratory rates make the child very tired from the increased work of breathing. In an infant there are very little reserves so the infant tires very quickly, especially when the work of sucking is added to the compromised respiratory state. To determine if the infant is hydrated the nurse should assess the skin turgor, palpate for a flat anterior fontanel, observe for moist mucus membranes and measure the urine output. The urine output should be 1 to 2ml/kg/hr. The electrolyte laboratory results will tell the nurse if the infant has an electrolyte imbalance, not a fluid imbalance. Assessing the lung sounds will not tell if the child is hydrated, only if the lungs are "wet" and fluid overloaded. The infant would also exhibit additional signs of respiratory distress if the lungs are fluid overloaded.

What is the earliest recognizable clinical manifestation(s) of CF? A. Meconium ileus B. History of poor intestinal absorption C. Foul-smelling, frothy, greasy stools D. Recurrent pneumonia and lung infections

A. Meconium ileus Rationale: The earliest clinical manifestation of CF is a meconium ileum, which is found in about 10% of children with CF. Clinical manifestations include abdominal distention, vomiting, failure to pass stools, and rapid development of dehydration.

What sign is indicative of respiratory distress in infants? A. Nasal flaring B. Respiratory rate of 55 breaths/min C. Irregular respiratory pattern D. Abdominal breathing

A. Nasal flaring Rationale: Infants have difficulty breathing through their mouths; therefore nasal flaring is usually accompanied by extra respiratory efforts. It also allows more air to enter as the nares flare.

A child presents to the health clinic with a temperature of 101.8°F (38.8°C), dysphagia, headache, and a sore, erythematous throat. Which collaborative intervention will the nurse complete first? A. Obtain a throat culture. B. Begin saltwater gargles. C. Assess the child for a rash. D. Administer oral antibiotics.

A. Obtain a throat culture. Rationale: A child presenting with fever, dysphagia, headache, and a sore, erythematous throat may have viral or bacterial pharyngitis. The nurse's first action should be to obtain a throat culture to determine if the child has a bacterial or viral infection. If the throat culture is negative, the child will not require antibiotics. If the culture is positive, the child will need antibiotics. Saltwater gargles will help relieve pain, but this is not a priority. Children with streptococcal pharyngitis may develop a sandpaper rash, but this information alone is not conclusive, and a throat culture should be obtained prior to administering antibiotics.

The nurse is caring for a 6-week-old with symptoms of irritability, nasal stuffiness, difficulty drinking and occasional vomiting. Which assessment finding produces important information regarding the medical and nursing treatment plan? A. Obtain testing for respiratory syncytial virus. B. Screen for the "allergic salute." C. Obtain vital signs to determine an infection. D. Draw a blood count to see if the client is septic.

A. Obtain testing for respiratory syncytial virus. Rationale: The symptoms presented are of acute nasopharyngitis. Many times this is viral in nature and can be common in the very young from respiratory syncytial virus (RSV). RSV is tested by obtaining nasal secretions and sending to the lab. A 6-week-old may rub his/her face but is too young for the "allergic salute," which is done to relieve itching and open nasal pathways. Vital signs can be helpful to note the beginning of an infectious process.

Which nursing diagnosis would best apply to a child with allergic rhinitis? A. Pain related to sinus edema and headache B. Ineffective tissue perfusion related to frequent nosebleeds C. Disturbed self-esteem related to inherited tendency for illness D. Risk for infection related to blocked eustachian tubes

A. Pain related to sinus edema and headache Rationale: Many children with allergic rhinitis develop sinus headaches from edema of the upper airway. In younger children the maxillary and ethmoid sinuses are involved. In children aged 10 years and older the frontal sinuses are also involved. The pain comes from mucosal swelling, decreased ciliary movement and a thickened nasal discharge. Nosebleeds are not common with either allergic rhinitis or sinusitis nor are either of these inherited. The eustachian tubes would cause symptoms of otitis, not of the nasal passage.

A nurse is caring for a 6-year-old client with cystic fibrosis. To enhance the child's nutritional status, which priority intervention should be included in the plan of care? A. Pancreatic enzyme supplementation with meals B. Provision of five to six small meals per day rather than three larger meals C. Total parenteral nutrition (TPN D. Magnesium, thiamine, and iron supplementation

A. Pancreatic enzyme supplementation with meals Rationale: Nearly 90% of clients with CF have pancreatic exocrine insufficiency and require oral pancreatic enzyme supplementation with meals. Frequent, small meals or TPN are not normally indicated. Vitamin supplements are required, but specific replacement of magnesium, thiamine, and iron is not typical.

A client has hypoxemia of pulmonary origin. What portion of arterial blood gas results is most useful in distinguishing between acute respiratory distress syndrome and acute respiratory failure? A. Partial pressure of arterial oxygen (PaO2) B. Partial pressure of arterial carbon dioxide (PaCO2) C. pH D. Bicarbonate (HCO3-)

A. Partial pressure of arterial oxygen (PaO2) Rationale: In acute respiratory failure, administering supplemental oxygen elevates the PaO2. In acute respiratory distress syndrome, elevation of the PaO2 requires positive end-expiratory pressure. In both situations, the PaCO2 is elevated and the pH and HCO3- are depressed.

The nurse is caring for a child with thickened pulmonary secretions. Which action(s) would the nurse use to assist the child breathe with less effort? Select all that apply. A. Perform chest physiotherapy B. Encourage oral fluids C. Avoid humidification of oxygen if oxygen is in use D. Assess pulse oximetry every 12 hours E. Observe for cyanosis and labored breathing every 12 hours

A. Perform chest physiotherapy B. Encourage oral fluids Rationale: Thickened pulmonary secretions occur with many respiratory disorders and illnesses. Encouraging oral liquids helps to thin the secretions so the child can easily cough them out. Chest physiotherapy is done to mobilize the secretions; therefore, the secretions are easier for the child to expectorate (spit out). Oxygen, if in use, should be humidified to avoid drying out the mucosa. The child should be observed for cyanosis and have pulse oximetry readings taken more frequently than every 12 or 24 hours. Often, the child is observed hourly or more.

The nurse is caring for a 6-month-old infant who has chronic apneic episodes. Which intervention should the nurse place in the plan of care? A. Place on a cardiopulmonary monitor and do frequent assessments. B. Teach the infant's parents how to perform infant cardiopulmonary resuscitation (CPR). C. Sit the infant upright in the infant seat to keep the airway open. D. Recommend surgical removal of tonsils and adenoids.

A. Place on a cardiopulmonary monitor and do frequent assessments. Rationale: The optimal treatments for infants, toddlers, and children with chronic apnea are hospitalization, frequent monitoring and observation, and parent education. The nurse should continuously monitor the infant on a cardiopulmonary monitor, frequently assess skin color, breathing patterns and effort of breathing, and assess for poor pharyngeal or laryngeal tone. The other actions do not include constant monitoring and assessments, which are crucial in treatment. The nurse is aware of apnea when the monitor alarms. Placing the child in an upright position will not stop the apneic episode. Teaching the parents about infant CPR is not a recommended activity for this case scenario. Although tonsillectomy and adenoidectomy are considered as a form of treatment for obstructive sleep apnea, recommending to parents these procedures is beyond the scope of practice of a nurse.

The nurse is assessing an adult client following a motor vehicle accident. The nurse observes that the client has an increased use of accessory muscles and is reporting chest pain and shortness of breath. The nurse should recognize the possibility of which condition? A. Pneumothorax B. Cardiac ischemia C. Acute bronchitis D. Aspiration

A. Pneumothorax Rationale: If the pneumothorax is large and the lung collapses totally, acute respiratory distress occurs. The client is anxious, has dyspnea and air hunger, has increased use of the accessory muscles, and may develop central cyanosis from severe hypoxemia. These symptoms are not definitive of pneumothorax, but because of the client's recent trauma they are inconsistent with cardiac ischemia, bronchitis, and aspiration.

A hospitalized child suddenly begins reporting "my chest hurts," is tachypneic, and has tachycardia. The nurse auscultates the lung sounds and finds absent breath sounds on one side. After notifying the health care provide what action would the nurse take first? A. Prepare for chest tube insertion B. Administer oxygen C. Obtain oxygen saturation measurement D. Prepare for mechanical ventilation

A. Prepare for chest tube insertion Rationale: A pneumothorax is a collection of air in the pleural space. Trapped air consumes space in the pleural cavity causing a partial or complete collapse. The priority symptom a nurse would assess is the decreased or absent lung sounds on the affected side. A pneumothorax can occur spontaneously in a healthy child or it can occur in a child with chronic lung disease, has been on a ventilator or has had thoracic surgery. Additional symptoms the child would experience would be chest pain, tachypnea, retractions, grunting, cyanosis and tachycardia. Many of these symptoms could be present with any child with an acute or chronic lung disease or respiratory distress, but the defining symptom is the absent breath sounds. The treatment for a pneumothoriax is with a chest tube so the priority action would be to gather supplies and prepare for the health care provider to insert a chest tube. Obtaining an oxygen saturation level measurement will only provide data, it will not help the child in distress. Oxygen may need to be administered, but with a pneumothorax it will be very ineffective. Mechanical ventilation would be a last resort and could actually make the situation worse if the lung was not reinflated.

A 5-year-old child is brought to the emergency department with copious drooling and a croaking sound on inspiration. Her mother states that the child is very agitated and only wants to sit upright. What should be the nurses first action in this situation? A. Prepare the intubation equipment and call the physician B. Examine the childs oropharynx and call the physician C. Obtain a throat culture for respiratory syncytial virus (RSV) D. Obtain vital signs and listen to breath sounds

A. Prepare the intubation equipment and call the physician Rationale: This child has symptoms of epiglottitis, is acutely ill, and requires emergency measures.

A client has been brought to the ED by the paramedics. The client is suspected of having acute respiratory distress syndrome (ARDS). What intervention should the nurse first anticipate? A. Preparing to assist with intubating the client B. Setting up oxygen at 5 L/minute by nasal cannula C. Performing deep suctioning D. Setting up a nebulizer to administer corticosteroids

A. Preparing to assist with intubating the client Rationale: A client who has ARDS usually requires intubation and mechanical ventilation. Oxygen by nasal cannula would likely be insufficient. Deep suctioning and nebulizers may be indicated, but the priority is to secure the airway.

A client has been discharged home after thoracic surgery. The home care nurse performs the initial visit and finds the client discouraged and saddened. The client states, "I am recovering so slowly. I really thought I would be better by now." What nursing action should the nurse prioritize? A. Provide emotional support to the client and family. B. Schedule a visit to the client's primary physician within 24 hours. C. Notify the physician that the client needs a referral to a psychiatrist. D. Place a referral for a social worker to visit the client.

A. Provide emotional support to the client and family. Rationale: The recovery process may take longer than the client had expected, and providing support to the client is an important task for the home care nurse. It is not necessary, based on this scenario, to schedule a visit with the physician within 24 hours, or to get a referral to a psychiatrist or a social worker.

The nurse is caring for a 7-year-old boy who has just had a tonsillectomy. Which intervention is least appropriate for this child? A. Providing fluids by straw B. Applying an ice collar C. Placing the child on his side D. Discouraging the child from coughing

A. Providing fluids by straw Rationale: Providing fluids by straw may cause trauma to the surgical site and should be avoided. Applying an ice collar, if ordered, helps relieve pain. Placing the child on his side, until he is fully awake, facilitates safe drainage of secretions. The child should be discouraged from coughing, clearing his throat, and blowing his nose to avoid trauma to the surgical site.

The nurse is assigned to care for a patient with COPD with hypoxemia and hypercapnia. When planning care for this patient, what does the nurse understand is the main goal of treatment? A. Providing sufficient oxygen to improve oxygenation B. Avoiding the use of oxygen to decrease the hypoxic drive C. Monitoring the pulse oximetry to assess need for early intervention when PCO2 levels rise D. Increasing pH

A. Providing sufficient oxygen to improve oxygenation Rationale: The main objective in treating patients with hypoxemia and hypercapnia is to give sufficient oxygen to improve oxygenation.

The nurse is caring for a child who has been admitted with a possible diagnosis of tuberculosis. Which laboratory/diagnostic tools would most likely be used to help diagnose this child? A. Purified protein derivative test B. Sweat sodium chloride test C. Blood culture and sensitivity D. Pulmonary functions test

A. Purified protein derivative test Rationale: Purified protein derivative tests are used to detect TB. Sweat sodium chloride tests are used for determining the diagnosis of cystic fibrosis. Blood culture and sensitivity is done to determine the causative agent as well as the anti-infective needed to treat an infection. Pulmonary function tests are diagnostic tools for the child with asthma and indicate the amount of obstruction in the bronchial airways, especially in the smallest airways of the lungs.

The nurse is assessing a client who, after an extensive surgical procedure, is at risk for developing acute respiratory distress syndrome (ARDS). The nurse assesses for which most common early sign of ARDS? A. Rapid onset of severe dyspnea B. Inspiratory crackles C. Bilateral wheezing D. Cyanosis

A. Rapid onset of severe dyspnea Rationale: The acute phase of ARDS is marked by a rapid onset of severe dyspnea that usually occurs less than 72 hours after the precipitating event.

In caring for the child with asthma, the nurse recognizes that bronchodilator medications are administered to children with asthma for which reason? A. Relief of acute symptoms B. Management of chronic pain C. To stabilize the cell membranes D. Prevention of mild symptoms

A. Relief of acute symptoms Rationale: Bronchodilators are used for quick relief of acute exacerbations of asthma symptoms. Mast cell stabilizers help to stabilize the cell membrane by preventing mast cells from releasing the chemical mediators that cause bronchospasm and mucous membrane inflammation. Leukotriene inhibitors are given by mouth along with other asthma medications for long-term control and prevention of mild, persistent asthma. Bronchodilators are not effective for pain.

A patient is being treated for status asthmaticus. What danger sign does the nurse observe that can indicate impending respiratory failure? A. Respiratory acidosis B. Respiratory alkalosis C. Metabolic acidosis D. Metabolic alkalosis

A. Respiratory acidosis Rationale: In status asthmaticus, increasing PaCO2 (to normal levels or levels indicating respiratory acidosis) is a danger sign signifying impending respiratory failure. Understanding the sequence of the pathophysiologic processes in status asthmaticus is important for understanding assessment findings. Respiratory alkalosis occurs initially because the patient hyperventilates and PaCO2 decreases. As the condition continues, air becomes trapped in the narrowed airways and carbon dioxide is retained, leading to respiratory acidosis.

In chronic obstructive pulmonary disease (COPD), decreased carbon dioxide elimination results in increased carbon dioxide tension in arterial blood, leading to which of the following acid-base imbalances? A. Respiratory acidosis B. Respiratory alkalosis C. Metabolic alkalosis D. Metabolic acidosis

A. Respiratory acidosis Rationale: Increased carbon dioxide tension in arterial blood leads to respiratory acidosis and chronic respiratory failure. In acute illness, worsening hypercapnia can lead to acute respiratory failure. The other acid-base imbalances would not correlate with COPD.

A nurse administers albuterol (Proventil), as ordered, to a client with emphysema. Which finding indicates that the drug is producing a therapeutic effect? A. Respiratory rate of 22 breaths/minute B. Dilated and reactive pupils C. Urine output of 40 ml/hour D. Heart rate of 100 beats/minute

A. Respiratory rate of 22 breaths/minute Rationale: In a client with emphysema, albuterol is used as a bronchodilator. A respiratory rate of 22 breaths/minute indicates that the drug has achieved its therapeutic effect because fewer respirations are required to achieve oxygenation. Albuterol has no effect on pupil reaction or urine output. It may cause a change in the heart rate, but this is an adverse, not therapeutic, effect.

During an assessment, a child exhibits an audible high-pitched inspiratory noise, a tripod stance and intercostal retractions. Using SBAR communication, the nurse notifies the health care provider and states which breath sounds that are congruent with the clinical presentation of the child? A. Respiratory stridor B. Wheezing in the bases C. Rales in the middle lobe D. Rhonchi throughout the lung

A. Respiratory stridor Rationale: Stridor is a high-pitched, readily audible inspiration noise that indicates an upper airway obstruction. The child presents in severe respiratory compromise and struggles to breathe. A wheeze is a high-pitched sound heard on auscultation, usually on expiration. It is due to obstruction in the lower trachea or bronchioles. Rales are crackling sounds heard on auscultation when the alveoli become fluid filled. Rhonchi is a snoring sound heard throughout the lung field when inflammation occurs.

Which classification of drugs is used to relieve an acute asthma episode? A. Short-acting beta2-adrenergic agonist B. Inhaled corticosteroids C. Leukotriene blockers D. Long-acting bronchodilators

A. Short-acting beta2-adrenergic agonist Rationale: Short-acting beta2-adrenergic agonist is the first medication administered. Later, systemic corticosteroids decrease airway inflammation in an acute asthma attack. They are given for short courses of 5-7 days.

A child requires supplemental oxygen therapy at 8 liters per minute. Which delivery device would the nurse most likely expect to be used? A. Simple mask B. Venturi mask C. Nasal cannula D. Oxygen hood

A. Simple mask Rationale: A simple mask would be used to deliver a flow rate of 8 liters per minute. A Venturi mask would be used to deliver a specific percentage of oxygen, from 24% to 50%. A nasal cannula would be used to deliver no more than 4 liters per minute. An oxygen hood requires a liter flow of 10 to 15 liters per minute.

A nurse is discussing asthma complications with a client and family. What complications should the nurse include in the teaching? Select all that apply. A. Status asthmaticus B. Respiratory failure C. Pertussis D. Atelectasis E. Thoracentesis

A. Status asthmaticus B. Respiratory failure D. Atelectasis Rationale: Complications of asthma may include status asthmaticus, respiratory failure, and atelectasis. Pertussis is not an asthma complication. Thoracentesis is a diagnostic procedure, not a complication.

The father of an infant calls the nurse to his son's room because he is making a strange noise. A diagnosis of laryngomalacia is made. What does the nurse expect to find on assessment? A. Stridor B. High-pitched cry C. Nasal congestion D. Spasmodic cough

A. Stridor Rationale: Stridor is usually present at birth but may begin as late as 2 months. Symptoms increase when the infant is supine or crying.

The nurse is caring for a 3-year-old girl with a respiratory disorder. The nurse anticipates the need for providing supplemental oxygen to the child when performing which action? A. Suctioning a tracheostomy tube B. Administering drugs with a nebulizer C. Providing tracheostomy care D. Suctioning with a bulb syringe

A. Suctioning a tracheostomy tube Rationale: Supplemental oxygenation may be necessary before, and is always performed after, suctioning a child with a tracheostomy tube. Providing tracheostomy care, administering drugs with a nebulizer, and suctioning with a bulb syringe do not require supplemental oxygen.

The nurse is assisting a physician with an endotracheal intubation for a client in respiratory failure. It is most important for the nurse to assess for: A. Symmetry of the client's chest expansion B. Tracheal cuff pressure set at 30 mm Hg C. Cool air humidified through the tube D. A scheduled time for deflation of the tracheal cuff

A. Symmetry of the client's chest expansion Rationale: Immediately after intubation, the nurse should check for symmetry of chest expansion. This is one finding that indicates successful endotracheal placement. The tracheal cuff pressure is set between 15 and 20 mm Hg. Routine deflation of the tracheal cuff is not recommended, because the client could then aspirate secretions during the period of deflation. Warm, high, humidified air is administered through the endotracheal tube.

The nurse expects the initial plan of care for a 9-month-old child with an acute otitis media infection to include: A. Symptomatic treatment and observation for 48 to 72 hours after diagnosis B. An oral antibiotic, such as amoxicillin, five times a day for 7 days C. Pneumococcal conjugate vaccine D. Myringotomy with tympanoplasty tubes

A. Symptomatic treatment and observation for 48 to 72 hours after diagnosis Rationale: Select children 6 months of age or older with acute otitis media are treated by initiating symptomatic treatment and observation for 48 to 72. hours.

A nurse is admitting a new client who has been admitted with a diagnosis of COPD exacerbation. How can the nurse best help the client achieve the goal of maintaining effective oxygenation? A. Teach the client strategies for promoting diaphragmatic breathing. B. Administer supplementary oxygen by simple face mask. C. Teach the client to perform airway suctioning. D. Assist the client in developing an appropriate exercise program.

A. Teach the client strategies for promoting diaphragmatic breathing. Rationale: The breathing pattern of most people with COPD is shallow, rapid, and inefficient; the more severe the disease, the more inefficient the breathing pattern. With practice, this type of upper chest breathing can be changed to diaphragmatic breathing, which reduces the respiratory rate, increases alveolar ventilation, and sometimes helps expel as much air as possible during expiration. Suctioning is not normally necessary in clients with COPD. Supplementary oxygen is not normally delivered by simple face mask and exercise may or may not be appropriate.

A mediastinal shift occurs in which type of chest disorder? A. Tension pneumothorax B. Traumatic pneumothorax C. Simple pneumothorax D. Cardiac tamponade

A. Tension pneumothorax Rationale: A tension pneumothorax causes the lung to collapse and the heart, the great vessels, and the trachea to shift toward the unaffected side of the chest (mediastinal shift). A traumatic pneumothorax occurs when air escapes from a laceration in the lung itself and enters the pleural space or enters the pleural space through a wound in the chest wall. A simple pneumothorax most commonly occurs as air enters the pleural space through the rupture of a bleb or a bronchopleural fistula. Cardiac tamponade is compression of the heart resulting from fluid or blood within the pericardial sac.

The nurse is assisting in the development of a plan of care for a child with asthma. In planning care, many goals would be appropriate for this child and/or family caregiver. Which two goals would be the highest priority for this child or family? A. The child will maintain a clear airway. B. The child will have adequate fluid intake. C. The child and family will connect with families living with the same diagnosis. D. The child and family will improve knowledge and understanding of varied pharmacologic options. E. The child will maintain adequate pain control.

A. The child will maintain a clear airway. B. The child will have adequate fluid intake. Rationale: Treatment and management of asthma centers around avoiding triggers and controlling inflammatory episodes. Keeping the airway open is always the priority (ABCs). The next physiologic need is adequate fluid intake. These are priorities over psychosocial considerations such as connecting with other families. Pain is not normally an issue. The family does not need to understand every available pharmacologic option. They need to understand the action plan for their child.

A client has been hospitalized for treatment of acute bacterial pneumonia. Which outcome indicates an improvement in the client's condition? A. The client has a partial pressure of arterial oxygen (PaO2) value of 90 mm Hg or higher. B. The client has a partial pressure of arterial carbon dioxide (PaCO2) value of 65 mm Hg or higher. C. The client exhibits restlessness and confusion. D. The client exhibits bronchial breath sounds over the affected area.

A. The client has a partial pressure of arterial oxygen (PaO2) value of 90 mm Hg or higher. Rationale: As the acute phase of bacterial pneumonia subsides, normal lung function returns and the PaO2 typically rises, reaching 85 to 100 mm Hg. A PaCO2 of 65 mm Hg or higher is above normal and indicates CO2 retention — common during the acute phase of pneumonia. Restlessness and confusion indicate hypoxia, not an improvement in the client's condition. Bronchial breath sounds over the affected area occur during the acute phase of pneumonia; later, the affected area should be clear on auscultation.

A nurse is planning the care of a client with bronchiectasis. What goal of care should the nurse prioritize? A. The client will successfully mobilize pulmonary secretions. B. The client will maintain an oxygen saturation level of 98%. C. The client's pulmonary blood pressure will decrease to within reference ranges. D. The client will resume prediagnosis level of function within 72 hours.

A. The client will successfully mobilize pulmonary secretions. Rationale: Nursing management focuses on alleviating symptoms and helping clients clear pulmonary secretions. Pulmonary pressures are not a central focus in the care of the client with bronchiectasis. Rapid resumption of prediagnosis function and oxygen saturation above 98% are unrealistic goals.

A client has tested positive for tuberculosis (TB). While providing client teaching, which information should the nurse prioritize? A. The importance of adhering closely to the prescribed medication regimen B. The disease being a lifelong, chronic condition that will affect activities of daily living (ADLs) C. TB being self-limiting but taking up to 2 years to resolve D. The need to work closely with the occupational and physical therapists

A. The importance of adhering closely to the prescribed medication regimen Rationale: Successful treatment of TB is highly dependent on careful adherence to the medication regimen. The disease is not self-limiting; occupational and physical therapy are not necessarily indicated. TB is curable.

The occupational health nurse is assessing an employee who has just had respiratory exposure to a toxin. What should the nurse assess? Select all that apply. A. Time frame of exposure B. Type of respiratory protection used C. Immunization status D. Breath sounds E. Intensity of exposure

A. Time frame of exposure B. Type of respiratory protection used D. Breath sounds E. Intensity of exposure Rationale: Key aspects of any assessment of clients with a potential occupational respiratory history include job and job activities, exposure levels, general hygiene, time frame of exposure, effectiveness of respiratory protection used, and direct versus indirect exposures. The client's current respiratory status would also be a priority. Occupational lung hazards are not normally influenced by immunizations.

A client who must begin oxygen therapy asks the nurse why this treatment is necessary? What would the nurse identify as the goals of oxygen therapy? Select all that apply. A. To provide adequate transport of oxygen in the blood B. To decrease the work of breathing C. To reduce stress on the myocardium D. To clear respiratory secretions E. To provide visual feedback to encourage the client to inhale slowly and deeply

A. To provide adequate transport of oxygen in the blood B. To decrease the work of breathing C. To reduce stress on the myocardium Rationale: Oxygen therapy is designed to provide adequate transport of oxygen in the blood while decreasing the work of breathing and reducing stress on the myocardium. Incentive spirometry is a respiratory modality that provides visual feedback to encourage the client to inhale slowly and deeply to maximize lung inflation and prevent or reduce atelectasis. A mini-nebulizer is used to help clear secretions.

Which of the following are risk factors for the development of chronic obstructive pulmonary disease (COPD)? Select all that apply. A. Tobacco smoke B. Occupational dust C. Air pollution D. Infection E. Second-hand smoke

A. Tobacco smoke B. Occupational dust C. Air pollution D. Infection E. Second-hand smoke Rationale: Risk factors for chronic obstructive pulmonary disease are tobacco smoke, environmental tobacco smoke, occupational dust and chemicals, indoor and outdoor air pollution, and infection.

Which comfort technique does a nurse teach to a client with pleurisy to assist with splinting the chest wall? A. Turn onto the affected side B. Use a prescribed analgesic C. Elevate the head of the bed D. Use a heat application

A. Turn onto the affected side Rationale: The nurse teaches the client to splint the chest wall by turning onto the affected side in order to reduce the stretching of the pleurae and decrease pain. The client will also take prescribed analgesic medications and use heat or cold applications to relieve pain, but these do not splint the chest wall. Another option for splinting the chest wall is for the client to use the hands or a pillow when coughing.

Which technique does the nurse suggest to a client with pleurisy while teaching about splinting the chest wall? A. Turn onto the affected side. B. Use a prescribed analgesic. C. Avoid using a pillow while splinting. D. Use a heat or cold application.

A. Turn onto the affected side. Rationale: The nurse teaches the client to splint the chest wall by turning onto the affected side. The nurse also instructs the client to take analgesic medications as prescribed and to use heat or cold applications to manage pain with inspiration. The client can also splint the chest wall with a pillow when coughing.

The nurse is caring for a client with suspected ARDS with a pO2 of 53. The client is placed on oxygen via face mask and the PO2 remains the same. What does the nurse recognize as a key characteristic of ARDS? A. Unresponsive arterial hypoxemia B. Diminished alveolar dilation C. Tachypnea D. Increased PaO2

A. Unresponsive arterial hypoxemia Rationale: Acute respiratory distress syndrome (ARDS) can be thought of as a spectrum of disease, from its milder form (acute lung injury) to its most severe form of fulminate, life-threatening ARDS. This clinical syndrome is characterized by a severe inflammatory process causing diffuse alveolar damage that results in sudden and progressive pulmonary edema, increasing bilateral infiltrates on chest x-ray, hypoxemia unresponsive to oxygen supplementation regardless of the amount of PEEP, and the absence of an elevated left atrial pressure.

A parent calls the "on call" line stating that her infant has had a bark-like cough for the past three nights. The parent states no fever or cold symptoms. Which suggestions may save a trip to the emergency department? Select all that apply. A. Use a cool mist humidifier in the infant's room. B. Take the infant into a steamy bathroom. C. Provide the infant cold oral fluids. D. Use the coolness of the night air. E. Assess throat for throat obstruction.

A. Use a cool mist humidifier in the infant's room. B. Take the infant into a steamy bathroom. D. Use the coolness of the night air. Rationale: The goal of the nurse is to provide suggestions which decrease the bark-like cough and relieve the bronchial constriction. Once this is accomplished, the infant can rest. Common suggestions are use of a cool mist humidifier, steamy bathroom, and coolness of the night air. Cold fluids may cause further spasm. The parent would not be instructed to assess the throat unless data suggested a problem in that location. More likely, the parent would be instructed to bring the infant to the emergency department.

Which vitamin is usually administered with isoniazid (INH) to prevent INH-associated peripheral neuropathy? A. Vitamin B6 B. Vitamin C C. Vitamin D D. Vitamin E

A. Vitamin B6 Rationale: Vitamin B6 (pyridoxine) is usually administered with INH to prevent INH-associated peripheral neuropathy. Vitamins C, D, and E are not appropriate.

Which assessment finding after tonsillectomy should be reported to the physician? A. Vomiting bright red blood B. Pain at surgical site C. Pain on swallowing D. The ability to only take small sips of liquids

A. Vomiting bright red blood Rationale: Vomiting bright red blood and swallowing frequently are signs of bleeding postoperatively and should be reported to the physician.

The nurse is admitting a child who is experiencing an asthma attack. Which clinical manifestation would likely be noted in this child? A. Wheezing B. Chest retractions C. Hoarseness D. Circumoral cyanosis

A. Wheezing Rationale: The onset of an attack can be very abrupt or can progress over several days, as evidenced by a dry hacking cough, wheezing (the sound of expired air being pushed through obstructed bronchioles), and difficulty breathing. Chest retractions can be seen when there is severe respiratory distress. Hoarseness is most commonly seen with illness such as laryngitis. Circumoral cyanosis is seen with children with poor perfusion, such as a child with a congenital heart defect.

The nurse is caring for a client who works construction with a focus on restoring and demolishing older buildings and who is diagnosed with pneumoconiosis. The nurse understands that the inflammation in the client's lungs is likely due to which substance? A. asbestos B. silica C. coal dust D. pollen

A. asbestos Rationale: Asbestosis is caused by inhalation of asbestos dust, which is frequently encountered during construction work, particularly when working with older buildings. Laws restrict asbestos use, but old materials still contain asbestos. Silicosis results from inhalation of silica dust and is seen in workers involved with mining, quarrying, stone cutting, and tunnel building. Inhalation of coal dust and other dusts may cause black lung disease. Pollen may cause an allergic reaction but is unlikely to cause pneumoconiosis.

A 5-year-old child is brought to the clinic by his father because the child developed a high fever over the past 2 to 3 hours. The nurse suspects epiglottitis based on which signs and symptoms? Select all that apply. A. difficulty speaking B. drooling C. sitting with neck extended D. frightened appearance E. cough

A. difficulty speaking B. drooling C. sitting with neck extended D. frightened appearance Rationale: Epiglottitis is manifested by a sudden onset of symptoms and high fever. The child has an overall toxic appearance and may refuse to speak or speak only in a very soft voice. The child may assume the characteristic position of sitting forward with the neck extended. Drooling may be present but a cough is usually absent. The child may appear frightened or anxious.

The nurse is collecting data on a child admitted with a respiratory concern. The nurse notes that the child is anxious and sitting forward with the neck extended to breathe. The signs the nurse noted indicate the child likely has: A. epiglottitis. B. asthma. C. cystic fibrosis. D. tuberculosis (TB).

A. epiglottitis. Rationale: The child with epiglottitis is very anxious and prefers to breathe by sitting forward with the neck extended. Immediate emergency attention is necessary. The child with asthma would have wheezing and distress trying to breathe. The child with cystic fibrosis would not have respiratory distress unless ill with respiratory infection. The drooling, leaning forward, and appearing distressed are not manifestations of TB.

What is a symptom of bacterial pharyngitis? A. fever B. rhinitis C. symptoms have gradual onset D. white blood cell (WBC) count in normal range

A. fever Rationale: Bacterial pharyngitis is most often caused by group A streptococcus. Fever is a symptom of bacterial pharyngitis. Other symptoms are an elevated WBC count, abrupt onset, headache, sore throat, abdominal discomfort, enlargement of tonsils, and firm cervical lymph nodes. It must be treated with an antibiotic. Penicillin is the drug of choice. Symptoms of rhinitis, a normal WBC count, and slow onset are indicative of viral pharyngitis.

A group of nursing students are reviewing information about variations in the anatomy of a child's respiratory tract structures in comparison to adults. The students demonstrate an understanding of the information when they describe the shape of the larynx in infants as: A. funnel. B. cylindrical. C. oval. D. spherical.

A. funnel. Rationale: In infants and children (younger than the age of 10 years), the cricoid cartilage is underdeveloped, resulting in laryngeal narrowing and a funnel-shaped larynx. In teenagers and adults, the larynx is cylindrical and fairly uniform in width.

A child has been admitted to the pediatric unit with pneumonia. The nurse is preparing to administer the prescribed medication to the child to help reduce the viscosity of the child's secretions. Which medication would the nurse most likely give? A. guaifenesin B. albuterol C. dextromethorphan D. ipratropium

A. guaifenesin Rationale: An expectorant, such as guaifenesin, reduces viscosity of thickened secretions by increasing respiratory tract fluid. Albuterol is a short-acting beta-adrenergic agonist that acts as a bronchodilator. Dextromethorphan is a cough suppressant. Ipratropium is an anticholinergic agent that acts as a bronchodilator.

A client with chronic obstructive pulmonary disease (COPD) is admitted to the medical-surgical unit. To help this client maintain a patent airway and achieve maximal gas exchange, the nurse should: A. instruct the client to drink at least 2 L of fluid daily. B. maintain the client on bed rest. C. administer anxiolytics, as ordered, to control anxiety. D. administer pain medication as ordered.

A. instruct the client to drink at least 2 L of fluid daily. Rationale: Mobilizing secretions is crucial to maintaining a patent airway and maximizing gas exchange in the client with COPD. Measures that help mobilize secretions include drinking 2 L of fluid daily, practicing controlled pursed-lip breathing, and engaging in moderate activity. Anxiolytics rarely are recommended for the client with COPD because they may cause sedation and subsequent infection from inadequate mobilization of secretions. Because COPD rarely causes pain, pain medication isn't indicated.

A child with a suspected airway obstruction is brought to the emergency room. The child produces a harsh, strident sound on inspiration (stridor). Where should the nurse anticipate the obstruction is most likely to be located? A. larynx B. lower trachea C. bronchioles D. pharynx

A. larynx Rationale: When the vibrations produced as air are forced past obstructions such as mucus in the nose or pharynx, the noise produced is a snoring sound (rhonchi). If the obstruction is at the base of the tongue or in the larynx, a harsher, strident sound on inspiration (stridor) occurs. If an obstruction is in the lower trachea or bronchioles, an expiratory whistle sound (wheezing) occurs.

A female nurse of childbearing ages caring for a 2-year-old child diagnosed with bacterial pneumonia. The child has been placed in a mist tent. In caring for the child, it is important for the nurse to: A. monitor the child regularly for signs of cyanosis. B. avoid contact with the mist. C. use contact transmission precautions. D. check for hyperthermia related to enclosure in the tent.

A. monitor the child regularly for signs of cyanosis. Rationale: In some treatment of bacterial pneumonia a croupette or mist tent is used. When a child is in a mist tent, the nurse must be constantly observed for cyanosis. Studies how that children have become cyanotic in mist tents and have suffer subsequent arrest, due to the lack of visibility while in the tent. Treatments for bacterial pneumonia do not carry risk for teratogenicity. Ribavirin is an antiviral drug that may be used to treat certain children with respiratory syncytial virus. It is administered as an inhalant by hood, mask, or tent and has a high risk for teratogenicity. In treating a client with bacterial pneumonia, the client may need to be placed on infection control precautions according to the policy of the health care facility; in most facilities, these will be droplet precautions. Although the nurse should look for hyperthermia related to the infection process, there is no risk for hyperthermia related to the child being enclosed in the tent.

A nurse assesses arterial blood gas results for a patient in acute respiratory failure (ARF). Which results are consistent with this disorder? A. pH 7.28, PaO2 50 mm Hg B. pH 7.46, PaO2 80 mm Hg C. pH 7.36, PaCO2 32 mm Hg D. pH 7.35, PaCO2 48 mm Hg

A. pH 7.28, PaO2 50 mm Hg Rationale: ARF is defined as a decrease in arterial oxygen tension (PaO2) to less than 60 mm Hg (hypoxemia) and an increase in arterial carbon dioxide tension (PaCO2) to >50 mm Hg (hypercapnia), with an arterial pH less than 7.35.

After teaching the parents of an 8-year-old girl with asthma about common allergens their child should avoid, the nurse determines that the parents need additional teaching when they identify what as a common allergen for asthma? A. shellfish B. indoor molds C. pet dander D. dust mites

A. shellfish Rationale: Eating shellfish is not a typical asthma trigger. Allergic reactions can occur with shellfish, but usually not an exacerbation of asthma. Indoor molds, pet dander, and dust mites are common asthma triggers.

The nurse identifies a nursing diagnosis of Ineffective airway clearance related to inflammation and copious thick secretions. What action is the priority? A. suctioning secretions from the airway B. administering oxygen as ordered C. monitoring oxygen saturation by pulse oximeter D. administering analgesics as ordered

A. suctioning secretions from the airway Rationale: The priority intervention is suctioning secretions to provide a patent airway. Administering oxygen as ordered, monitoring oxygen saturation by pulse oximeter, and administering analgesics as ordered would be secondary interventions.

The nurse is preparing to administer albuterol to a 14-year-old client for the first time. Prior to administration, which adverse reaction is priority for the nurse to educate the client? A. tachycardia B. hypoactivity C. bronchial muscle relaxation D. increased appetite

A. tachycardia Rationale: Adverse reactions of albuterol, a bronchodilator, include tachycardia, nervousness, tremors, hyperactivity, malaise, palpitations, increased appetite, hypokalemia, and muscle cramps. The expected action of albuterol is to relax bronchial, uterine, and vascular smooth muscle by stimulating beta-2 receptors. While tachycardia and increased appetite are both adverse reactions, tachycardia happens abruptly following the first dose and can be alarming for clients. It is a priority for the nurse to provide education on this over a slower, less concerning change.

The nurse is caring for a 10-year-old girl with allergic rhinitis. Which intervention helps prevent secondary bacterial infection? A. using normal saline nasal washes B. teaching parents how to avoid allergens C. discussing anti-inflammatory nasal sprays D. educating parents about oral antihistamines

A. using normal saline nasal washes Rationale: Using nasal washes to improve air flow will help prevent secondary bacterial infection by preventing the mucus from becoming thick and immobile. Teaching parents how to avoid allergens such as tobacco smoke, dust mites, and molds helps prevent recurrence of allergic rhinitis. Discussing anti-inflammatory nasal sprays and teaching parents about using oral antihistamines would help in prevention and treatment of the disorder.

A nurse is giving a speech addressing "Communicable Diseases of Winter" to a large group of volunteer women, most of whom are older than 60 years. What preventive measures should the nurse recommend to these women, who are at the risk of pneumococcal and influenza infections? Select all that apply. A. vaccinations B. hand antisepsis C. incentive spirometry D. prescribed opioids

A. vaccinations B. hand antisepsis Rationale: A powerful weapon against the spread of communicable disease is effective and frequent handwashing. The pneumococcal vaccine provides specific prevention against pneumococcal pneumonia and other infections caused by S. pneumoniae.

A nurse is giving a speech about communicable diseases of winter to a large group of volunteer women, most of whom are older than 60 years. What preventive measure(s) should the nurse recommend to these women, who are at risk of pneumococcal and influenza infections? Select all that apply. A. vaccinations B. hand antisepsis C. annual echocardiogram D. prescribed opioids E. smoking cessation

A. vaccinations B. hand antisepsis E. smoking cessation Rationale: A powerful weapon against the spread of communicable disease is effective and frequent handwashing. The pneumococcal vaccine provides specific prevention against pneumococcal pneumonia and other infections caused by S. pneumoniae. The nurse should encourage smoking cessation as a preventative measure against respiratory illness. Annual echocardiograms and prescribed opioids do not reduce the risk of pneumococcal and influenza infections.

The nurse hears wheezing when auscultating a 4-year-old. Which of the following conditions would the nurse most likely rule out based on the assessment findings? A) Bronchiolitis B) Asthma C) Influenza D) Cystic fibrosis

B) Asthma

A community health nurse is conducting a parenting class on respiratory syncytial virus (RSV). What statement made by a parent indicates that the teaching has been successful? A. "RSV season occurs primarily April through September." B. "Exposure to second- or third-hand smoke increases the risk for developing RSV." C. "Infants are less affected by RSV than older children." D. "Early initiation of antibiotics can lessen the severity of the infection."

B. "Exposure to second- or third-hand smoke increases the risk for developing RSV." Rationale: An infant exposed to second- or third-hand smoke is at risk for developing respiratory syncytial virus (RSV). RSV season runs from September through April. Current treatment recommendations for RSV do not include antibiotics. Infants are susceptible to RSV much more than older children.

A client with chronic obstructive pulmonary disease (COPD) expresses a desire to quit smoking. The first appropriate response from the nurse is: A. "Nicotine patches would be appropriate for you." B. "Have you tried to quit smoking before?" C. "I can refer you to the American Lung Association." D. "Many options are available for you."

B. "Have you tried to quit smoking before?" Rationale: All the options are appropriate statements; however, the nurse needs to assess the client's statement further. Assessment data include information about previous attempts to quit smoking.

The nurse is auscultating the lungs of a lethargic, irritable 6-year-old boy and hears wheezing. The nurse will most likely include which teaching point if the child is suspected of having asthma? A. "I'm going to have the respiratory therapist get some of the mucus from your lungs." B. "I'm going to have this hospital worker take a picture of your lungs." C. "We're going to go take a look at your lungs to see if there are any sores on them." D. "I'm going to hold your hand while the phlebotomist gets blood from your arm."

B. "I'm going to have this hospital worker take a picture of your lungs." Rationale: The nurse should teach the child using terms a 6-year-old will understand. A chest x-ray is usually ordered for the assessment of asthma to check for hyperventilation. A sputum culture is indicated for pneumonia, cystic fibrosis, and tuberculosis; fluoroscopy is used to identify masses or abscesses as with pneumonia; and the sweat chloride test is indicated for cystic fibrosis.

The nurse is providing discharge teaching to the parents of a child who had a tonsillectomy. Which statement(s) by the parents indicate learning has occurred? Select all that apply. A. "Warm soup should be easy to swallow and will help with controlling the pain." B. "If our child starts swallowing a lot, we may need to call the health care provider." C. "Milkshakes should be drunk with straws so that not too much is swallowed at a time." D. "Fluids are very important. Our child loves popsicles so we will get a variety of flavors, except cherry and strawberry." E. "We can use an ice collar on the throat as long as we do not leave it on too long at a time."

B. "If our child starts swallowing a lot, we may need to call the health care provider." D. "Fluids are very important. Our child loves popsicles so we will get a variety of flavors, except cherry and strawberry." E. "We can use an ice collar on the throat as long as we do not leave it on too long at a time." Rationale: Excessive swallowing is an indication of bleeding and should be reported to the health care provider. Fluids are very important, but red fluids should be avoided as they can be mistaken for blood. Ice collars can be used for pain and to decrease bleeding, but they should not be left on the throat for extended periods. Hot or warm liquids and the use of straws would cause bleeding to occur and should be avoided.

The nurse is providing education to a client newly diagnosed with asthma. Which statement by the parents indicates additional teaching is needed? A. "We will keep an albuterol inhaler with our child at all times." B. "It is okay for our child to do chores such as sweeping the floor." C. "We will both enroll in smoking cessation classes." D. "Our family dog will need to go live with a grandparent."

B. "It is okay for our child to do chores such as sweeping the floor." Rationale: Sweeping the floor can trigger a child's asthma by making environmental allergens and irritants airborne, causing upper respiratory infections. The nurse will intervene if the parents make this statement. An inhaler should be with the child at all times in case of an asthma attack. Smoke and pet allergens can trigger an attack and exposure should be avoided. Other triggers are exercise, weather changes, air pollution, foods, and certain medications.

A client's severe asthma has necessitated the use of a long-acting beta2-agonist (LABA). Which of the client's statements suggests a correct understanding of this medication? A. "This drug may make my heart beat slower." B. "This drug is particularly good at preventing asthma attacks during exercise." C. "I'll make sure to use this each time I feel an asthma attack coming on." D. "I understand that this drug is less effective at controlling night-time symptoms."

B. "This drug is particularly good at preventing asthma attacks during exercise." Rationale: LABAs are effective in the prevention of exercise-induced asthma. They are also used with anti-inflammatory medications to control asthma symptoms, particularly those that occur during the night. LABAs are not indicated for immediate relief of symptoms. are not used for management of acute asthma symptoms. Tachycardia, not bradycardia, is a potential adverse effect of this medication.

A new employee asks the occupational health nurse about measures to prevent inhalation exposure to toxic substances. What should the nurse recommend? A. "Position a fan blowing toxic substances away from you to prevent you from being exposed." B. "Wear protective attire and devices when working with a toxic substance." C. "Make sure that you keep your immunizations up to date to prevent respiratory diseases resulting from toxins." D. "Always wear a disposable paper face mask when you are working with inhalable toxins."

B. "Wear protective attire and devices when working with a toxic substance." Rationale: When working with toxic substances, the employee must wear or use protective devices such as face masks, hoods, or industrial respirators. Immunizations do not confer protection from toxins and a paper mask is normally insufficient protection. Never position a fan directly blowing on the toxic substance as it will disperse the fumes throughout the area.

A patient who wears contact lenses is to be placed on rifampin for tuberculosis therapy. What should the nurse tell the patient? A. "Only wear your contact lenses during the day and take them out in the evening before bed." B. "You should switch to wearing your glasses while taking this medication." C. "The physician can give you eye drops to prevent any problems." D. "There are no significant problems with wearing contact lenses."

B. "You should switch to wearing your glasses while taking this medication." Rationale: The nurse informs the patient that rifampin may discolor contact lenses and that the patient may want to wear eyeglasses during treatment.

What would the critical care nurse recognize as a condition that may indicate a client's need to have a tracheostomy? A. A client has a respiratory rate of 10 breaths per minute. B. A client requires permanent ventilation. C. A client exhibits symptoms of dyspnea. D. A client has respiratory acidosis.

B. A client requires permanent ventilation. Rationale: A tracheostomy permits long-term use of mechanical ventilation to prevent aspiration of oral and gastric secretions in the unconscious or paralyzed client. Indications for a tracheostomy do not include a respiratory rate of 10 breaths per minute, symptoms of dyspnea, or respiratory acidosis.

A 67-year-old client with severe bilateral arthritis of the hands has been diagnosed with chronic obstructive pulmonary disease. Which inhalation delivery method is best for the bronchodilator ordered for this client? A. A conventional pressurized metered-dose inhaler (pMDI) B. A small-volume nebulizer (SVN) C. A breath-actuated pressurized metered-dose inhaler (pMDI) D. A dry-powder inhaler (DPI)

B. A small-volume nebulizer (SVN) Rationale: A SVN is the best choice for the client with severe arthritis in both hands. The SVN is a machine that mixes the medication and turns it into a mist. The client then uses a handheld apparatus or mask and just breathes in the treatment. Any pMDI requires dexterity between inspiration and the mechanics of the inhaler to be effective. Although DPIs rely solely on inspiration for medication delivery, they still require the user to press a button or lever to dispense the medication. Poor inhaler technique has been linked to a lack of symptom control.

A client with thoracic trauma is admitted to the ICU. The nurse notes the client's chest and neck are swollen and there is a crackling sensation when palpated. The nurse consequently identifies the presence of subcutaneous emphysema. If this condition becomes severe and threatens airway patency, what intervention is indicated? A. A chest tube B. A tracheostomy C. An endotracheal tube D. A feeding tube

B. A tracheostomy Rationale: In severe cases in which there is widespread subcutaneous emphysema, a tracheostomy is indicated if airway patency is threatened by pressure of the trapped air on the trachea. The other listed tubes would neither resolve the subcutaneous emphysema nor the consequent airway constriction.

The nurse caring for a 2-year-old near-drowning victim monitors for what possible complication? A. Atelectasis B. Acute respiratory distress syndrome C. Metabolic alkalosis D. Respiratory acidosis

B. Acute respiratory distress syndrome Rationale: Factors associated with the development of acute respiratory distress syndrome include aspiration related to near drowning or vomiting; drug ingestion/overdose; hematologic disorders such as disseminated intravascular coagulation or massive transfusions; direct damage to the lungs through prolonged smoke inhalation or other corrosive substances; localized lung infection; metabolic disorders such as pancreatitis or uremia; shock; trauma such as chest contusions, multiple fractures, or head injury; any major surgery; embolism; and septicemia. The nurse would not monitor for atelectasis, metabolic alkalosis, or respiratory acidosis in this scenario.

A hospital has been the site of an increased incidence of hospital-acquired pneumonia (HAP). What is an important measure for the prevention of HAP? A. Administration of prophylactic antibiotics B. Administration of pneumococcal vaccine to vulnerable individuals C. Obtaining culture and sensitivity swabs from all newly admitted clients D. Administration of antiretroviral medications to clients over age 65

B. Administration of pneumococcal vaccine to vulnerable individuals Rationale: Pneumococcal vaccination reduces the incidence of pneumonia, hospitalizations for cardiac conditions, and deaths in the general older adult population. A one-time vaccination of pneumococcal polysaccharide vaccine (PPSV) is recommended for all clients 65 years of age or older and those with chronic diseases. Antibiotics are not given on a preventative basis and antiretroviral medications do not affect the most common causative microorganisms. Culture and sensitivity testing by swabbing is not performed for pneumonia since the microorganisms are found in sputum.

A nurse is assessing a client who is suspected of having bronchiectasis. The nurse should consider which of the following potential causes? Select all that apply. A. Pulmonary hypertension B. Airway obstruction C. Pulmonary infections D. Genetic disorders E. Atelectasis

B. Airway obstruction C. Pulmonary infections D. Genetic disorders Rationale: Bronchiectasis is a chronic, irreversible dilation of the bronchi and bronchioles. Under the new definition of COPD, it is considered a disease process separate from COPD. Bronchiectasis may be caused by a variety of conditions, including airway obstruction, diffuse airway injury, pulmonary infections and obstruction of the bronchus or complications of long-term pulmonary infections, or genetic disorders such as cystic fibrosis. Bronchiectasis is not caused by pulmonary hypertension or atelectasis.

A group of nursing students are reviewing the medications used to treat asthma. The students demonstrate understanding of the information when they identify which agent as appropriate for an acute episode of bronchospasm? A. Salmeterol B. Albuterol C. Ipratropium D. Cromolyn

B. Albuterol Rationale: Albuterol is a short-acting β2-adrenergic agonist that is used for treatment of acute bronchospasm. Salmeterol is a long-acting β2-adrenergic agonist used for long-term control or exercise-induced asthma. Ipratropium is an anticholinergic agent used as an adjunct to β2- adrenergic agonists for treatment of bronchospasm. Cromolyn is a mast cell stabilizer used prophylactically but not to relieve bronchospasm during an acute wheezing episode.

An interdisciplinary team is planning the care of a client with bronchiectasis. What aspects of care should the nurse anticipate? Select all that apply. A. Occupational therapy B. Antimicrobial therapy C. Positive pressure isolation D. Chest physiotherapy E. Smoking cessation

B. Antimicrobial therapy D. Chest physiotherapy E. Smoking cessation Rationale: Chest physiotherapy, antibiotics, and smoking cessation are cornerstones of the care of clients with bronchiectasis. Occupational therapy and isolation are not normally indicated.

A nurse is assigned to a client diagnosed with emphysema. In prioritizing the care for the shift, which intervention should the nurse choose first? A. Refer to respiratory therapy if labored breathing persists. B. Apply oxygen therapy as ordered. C. Monitor vital signs every 2 hours, including respiratory rate and characteristics. D. Educate regarding smoking cessation.

B. Apply oxygen therapy as ordered. Rationale: Oxygen will correct the hypoxemia. Careful observation of the liter flow or the percentage administered and its effect on the patient is important. These clients generally require low flow oxygen rates of 1-2 L/min. Monitor and titrate to achieve desired PaO2. Periodic arterial blood gases and pulse oximetry help evaluate adequacy of oxygenation. The priority action is not referral to the respiratory therapist. Vital signs monitoring is important, but not the first action. Education is part of the shift, but not the priority.

A perioperative nurse is caring for a postoperative client. The client has a shallow respiratory pattern and is reluctant to cough or to begin mobilizing. The nurse should address the client's increased risk for what complication? A. Acute respiratory distress syndrome (ARDS) B. Atelectasis C. Aspiration D. Pulmonary embolism

B. Atelectasis Rationale: A shallow, monotonous respiratory pattern coupled with immobility places the client at an increased risk of developing atelectasis. These specific factors are less likely to result in pulmonary embolism or aspiration. ARDS involves an exaggerated inflammatory response and does not normally result from factors such as immobility and shallow breathing.

The nurse is obtaining data from a client with a respiratory disorder. Which information would be considered a part of the functional assessment and would assist in the diagnosis of an occupational lung disease? A. Cough and dyspnea B. Black-streaked sputum C. Tenacious secretions D. Barrel chest

B. Black-streaked sputum Rationale: A functional assessment provides data on the lifestyle, living environment, and work environment of the client, which can contribute to lung disorders. A black-tinged sputum is suggestive of prolonged exposure to coal dust. Cough, dyspnea, and tenacious secretions are vague respiratory symptoms that are not specific to occupational lung disease. The presence of barrel chest is indicative of trapped oxygen in the lungs over a prolonged period of time.

The nurse is caring for a client suspected of having acute respiratory distress syndrome (ARDS). What is the most likely diagnostic test ordered in the early stages of this disease to differentiate the client's symptoms from those of a cardiac etiology? A. Carboxyhemoglobin level B. Brain natriuretic peptide (BNP) level C. C-reactive protein (CRP) level D. Complete blood count

B. Brain natriuretic peptide (BNP) level Rationale: Common diagnostic tests performed for clients with potential ARDS include plasma brain natriuretic peptide (BNP) levels, echocardiography, and pulmonary artery catheterization. The BNP level is helpful in distinguishing ARDS from cardiogenic pulmonary edema. The carboxyhemoglobin level will be increased in a client with an inhalation injury, which commonly progresses into ARDS. CRP and CBC levels do not help differentiate from a cardiac problem.

A nurse is reviewing the pathophysiology of cystic fibrosis (CF) in anticipation of a new admission. The nurse should identify what characteristic aspects of CF? A. Alveolar mucus plugging, infection, and eventual bronchiectasis B. Bronchial mucus plugging, inflammation, and eventual bronchiectasis C. Atelectasis, infection, and eventual COPD D. Bronchial mucus plugging, infection, and eventual COPD

B. Bronchial mucus plugging, inflammation, and eventual bronchiectasis Rationale: The hallmark pathology of CF is bronchial mucus plugging, inflammation, and eventual bronchiectasis. Commonly, the bronchiectasis begins in the upper lobes and progresses to involve all lobes. Infection, atelectasis, and COPD are not hallmark pathologies of CF.

What is the best nursing response to the parent of a child with asthma who asks if his child can still participate in sports? A. Children with asthma are usually restricted from physical activities B. Children can usually play any type of sport if their asthma is well controlled C. Avoid swimming because breathing underwater is dangerous for people with asthma D. Even with good asthma control, I would advice limiting the child to one athletic activity per school year

B. Children can usually play any type of sport if their asthma is well controlled Rationale: Sports that do not require sustained exertion, such as gymnastics, baseball, weight lifting, are well tolerated. Children can usually play any type of sport if their asthma is well controlled.

A group of nursing students are reviewing information about the variations in respiratory anatomy and physiology in children in comparison to adults. The students demonstrate understanding of the information when they identify which finding? A. Children's demand for oxygen is lower than that of adults. B. Children develop hypoxemia more rapidly than adults do. C. An increase in oxygen saturation leads to a much larger decrease in pO2. D. Children's bronchi are wider in diameter than those of an adult.

B. Children develop hypoxemia more rapidly than adults do. Rationale: Children develop hypoxemia more rapidly than adults do because they have a significantly higher metabolic rate and faster resting respiratory rates than adults do, which leads to a higher demand for oxygen. A smaller decrease in oxygen saturation reflects a disproportionately much larger decrease in pO2. The bronchi in children are narrower than in adults, placing them at higher risk for lower airway obstruction.

A nurse is developing the teaching portion of a care plan for a client with COPD. What would be the most important component for the nurse to emphasize? A. Smoking up to three cigarettes weekly is generally allowable. B. Chronic inhalation of indoor toxins can cause lung damage. C. Minor respiratory infections are considered to be self-limited and are not treated with medication. D. Activities of daily living (ADLs) should be clustered in the early morning hours.

B. Chronic inhalation of indoor toxins can cause lung damage. Rationale: Environmental risk factors for COPD include prolonged and intense exposure to occupational dusts and chemicals, indoor air pollution, and outdoor air pollution. Smoking cessation should be taught to all clients who are currently smoking. Minor respiratory infections that are of no consequence to the person with normal lungs can produce fatal disturbances in the lungs of the person with emphysema. ADLs should be paced throughout the day to permit clients to perform these without excessive distress.

The infant with bronchopulmonary dysplasia (BPD) who has RSV bronchiolitis is a candidate for which treatment? A. Pancreatic enzymes B. Cool humidified oxygen C. Erythromycin intravenously D. Intermittent positive pressure ventilation

B. Cool humidified oxygen Rationale: Humidified oxygen is delivered if the oxygen saturation level drops to less than 90%.

A patient is admitted to the hospital with pulmonary arterial hypertension. What assessment finding by the nurse is a significant finding for this patient? A. Ascites B. Dyspnea C. Hypertension D. Syncope

B. Dyspnea Rationale: Dyspnea, the main symptom of PH, occurs at first with exertion and eventually at rest. Substernal chest pain also is common. Other signs and symptoms include weakness, fatigue, syncope, occasional hemoptysis, and signs of right-sided heart failure (peripheral edema, ascites, distended neck veins, liver engorgement, crackles, heart murmur). Anorexia and abdominal pain in the right upper quadrant may also occur.

An asthma nurse educator is working with a group of adolescent asthma clients. What intervention is most likely to prevent asthma exacerbations among these clients? A. Encouraging clients to carry a corticosteroid rescue inhaler at all times B. Educating clients about recognizing and avoiding asthma triggers C. Teaching clients to utilize alternative therapies in asthma management D. Ensuring that clients keep their immunizations up to date

B. Educating clients about recognizing and avoiding asthma triggers Rationale: Asthma exacerbations are best managed by early treatment and education, including the use of written action plans as part of any overall effort to educate clients about self-management techniques, especially those with moderate or severe persistent asthma or with a history of severe exacerbations. Corticosteroids are not used as rescue inhalers. Alternative therapies are not normally a high priority, though their use may be appropriate in some cases. Immunizations should be kept up to date, but this does not necessarily prevent asthma exacerbations.

Which type of croup is always considered a medical emergency? A. Laryngitis B. Epiglottitis C. Spasmodic croup D. Laryngotracheobronchitis (LTB)

B. Epiglottitis Rationale: Epiglottitis is always a medical emergency that requires antibiotics and airway support for treatment.

The nurse is correct to identify which group of symptoms as present with acute laryngotracheobronchitis? A. Wheezing and a moist cough B. Fever and labored respirations C. Crackles in lung fields with cyanosis D. Hypoxia with shallow respirations

B. Fever and labored respirations Rationale: Acute laryngotracheobronchitis presents with hoarseness and a barking cough with fever reaching 104°F to 105°F (40°C to 40.6°C) As the disease progresses, marked laryngeal edema occurs with dyspnea, a rapid pulse and cyanosis. Stridor is heard in the lung fields.

An asthma educator is teaching a client newly diagnosed with asthma and the family about the use of a peak flow meter. The educator should teach the client that a peak flow meter measures highest airflow during which type of breath? A. Forced inspiration B. Forced expiration C. Normal inspiration D. Normal expiration

B. Forced expiration Rationale: Peak flow meters measure the highest airflow during a forced expiration.

A nurse consulting with a nutrition specialist knows it's important to consider a special diet for a client with chronic obstructive pulmonary disease (COPD). Which diet is appropriate for this client? A. Full-liquid B. High-protein C. 1,800-calorie ADA D. Low-fat

B. High-protein Rationale: Breathing is more difficult for clients with COPD, and increased metabolic demand puts them at risk for nutritional deficiencies. These clients must have a high intake of protein for increased calorie consumption. Full liquids, 1,800-calorie ADA, and low-fat diets aren't appropriate for a client with COPD.

The nurse is caring for a client who is scheduled to have a thoracotomy. When planning preoperative teaching, what information should the nurse communicate to the client? A. How to milk the chest tubing B. How to splint the incision when coughing C. How to take prophylactic antibiotics correctly D. How to manage the need for fluid restriction

B. How to splint the incision when coughing Rationale: Prior to thoracotomy, the nurse educates the client about how to splint the incision with the hands, a pillow, or a folded towel. The client is not taught how to milk the chest tubing because this is performed by the nurse. Prophylactic antibiotics are not normally used and fluid restriction is not indicated following thoracotomy.

A nurse is preparing a teaching plan for the family of a child with allergic rhinitis. When describing the immune reaction that occurs, the nurse would identify the role of which immunoglobulin? A. IgA B. IgE C. IgG D. IgM

B. IgE Rationale: The immunoglobulin involved in the immune response associated with allergic rhinitis is IgE. IgA, IgG, and IgM are not involved in this response.

The nurse has instructed a client on how to perform pursed-lip breathing. The nurse recognizes the purpose of this type of breathing is to accomplish which result? A. Promote more efficient and controlled ventilation and to decrease the work of breathing B. Improve oxygen transport; induce a slow, deep breathing pattern; and assist the client to control breathing C. Promote the strengthening of the client's diaphragm D. Promote the client's ability to take in oxygen

B. Improve oxygen transport; induce a slow, deep breathing pattern; and assist the client to control breathing Rationale: Pursed-lip breathing, which improves oxygen transport, helps induce a slow, deep breathing pattern and assists the client to control breathing, even during periods of stress. This type of breathing helps prevent airway collapse secondary to loss of lung elasticity in emphysema.

Which measure may increase complications for a client with COPD? A. Administration of antibiotics B. Increased oxygen supply C. Administration of antitussive agents D. Decreased oxygen supply

B. Increased oxygen supply Rationale: Administering too much oxygen can result in the retention of carbon dioxide. Clients with alveolar hypoventilation cannot increase ventilation to adjust for this increased load, and hypercapnia occurs. All the other measures aim to prevent complications.

In a client with amyotrophic lateral sclerosis (ALS) and respiratory distress, which finding is the earliest sign of reduced oxygenation? A. Decreased heart rate B. Increased restlessness C. Increased blood pressure D. Decreased level of consciousness (LOC)

B. Increased restlessness Rationale: In ALS, an early sign of respiratory distress is increased restlessness, which results from inadequate oxygen flow to the brain. As the body tries to compensate for inadequate oxygenation, the heart rate increases and blood pressure drops. A decreased LOC is a later sign of poor tissue oxygenation in a client with respiratory distress.

A nursing student understands the importance of the psychosocial aspects of disease processes. When working with a patient with COPD, the student would rank which of the following nursing diagnoses as the MOST important when analyzing the psychosocial effects? A. Disturbed sleep pattern related to cough B. Ineffective coping related to anxiety C. High risk for ineffective therapeutic regimen management related to lack of knowledge D. Activity intolerance related to fatigue

B. Ineffective coping related to anxiety Rationale: Any factor that interferes with normal breathing quite naturally induces anxiety, depression, and changes in behavior. Constant shortness of breath and fatigue may make the patient irritable and apprehensive to the point of panic. Although the other choices are correct, the most important psychosocial nursing diagnosis for a patient with COPD is ineffective coping related to a high level of anxiety.

The parent of a toddler calls the nurse, asking about croup. What is a distinguishing manifestation of spasmodic croup? A. Wheezing is heard audibly B. It has a harsh, barky cough C. It is bacterial in nature D. The child has a high fever

B. It has a harsh, barky cough Rationale: Spasmodic croup is viral in origin; is usually preceded by several days of symptoms of upper respiratory tract infections; often begins at night; and is marked by hard, metallic, bark cough; sore throat; inspiratory stridor; and hoarseness.

A nursing instructor is preparing a class on chronic lung disease. What information would the instructor include when describing this disorder? A. It is a result of cystic fibrosis. B. It is seen most commonly in premature infants. C. It typically affects females more often than males. D. It is characterized by bradypnea.

B. It is seen most commonly in premature infants. Rationale: Chronic lung disease, formerly known as bronchopulmonary dysplasia, is often diagnosed in infants who have experienced respiratory distress syndrome, most commonly seen in premature infants. Male gender is a risk factor for development. Tachypnea and increased work of breathing are characteristic of chronic lung disease.

A client is brought to the emergency department by ambulance after a motor vehicle accident in which the client received blunt trauma to the chest. The client is in acute respiratory failure, intubated, and transferred to the intensive care unit (ICU). Which assessment parameters should the nurse monitor most closely? Select all that apply. A. Coping B. Level of consciousness C. Oral intake D. Arterial blood gases E. Vital signs

B. Level of consciousness D. Arterial blood gases E. Vital signs Rationale: Trauma clients are usually treated in the ICU. The nurse assesses the client's respiratory status by monitoring the level of responsiveness, arterial blood gases, pulse oximetry, and vital signs. Oral intake and coping are not immediate priorities during the acute stage of treatment but would become more important later during recovery.

A client is receiving thrombolytic therapy for the treatment of pulmonary emboli. What is the best way for the nurse to assess the client's oxygenation status at the bedside? A. Obtain serial ABG samples. B. Monitor pulse oximetry readings. C. Perform chest auscultation. D. Monitor incentive spirometry volumes.

B. Monitor pulse oximetry readings. Rationale: The nurse assesses the client with pulmonary emboli frequently for signs of hypoxemia and monitors the pulse oximetry values to evaluate the effectiveness of the oxygen therapy. ABGs are accurate indicators of oxygenation status, but are not analyzed at the bedside. PFTs and incentive spirometry volumes do not accurately reveal oxygenation status.

What statement is the most accurate regarding the structure and function of the newborn's respiratory system? A. The diameter of the child's trachea is the same as that of adults. B. Most infants are nasal breathers rather than mouth breathers. C. The respiratory tract in the child is fully developed by age 2. D. Infants and young children have smaller tongues in proportion to their mouths.

B. Most infants are nasal breathers rather than mouth breathers. Rationale: Newborns are obligatory nose breathers until at least 4 weeks of age. The diameter of the infant and child's trachea is about the size of the child's little finger. The respiratory tract grows and changes until the child is about 12 years of age. During the first 5 years of life, infants and young children have larger tongues in proportion to their mouths.

Which statement indicates that a parent of a toddler needs more education about preventing foreign body aspiration? A. I keep objects with small parts out of reach B. My toddler loves to play with balloons C. I won't permit my child to have peanuts D. I never leave coins where my child could get them

B. My toddler loves to play with balloons Rationale: Latex balloons account for a significant number of deaths from aspiration every year.

The nurse is caring for a client who is experiencing mild shortness of breath during the immediate postoperative period, with oxygen saturation readings between 89% and 91%. Which method of oxygen delivery is most appropriate for the client's needs? A. Nonrebreathing mask B. Nasal cannula C. Venturi mask D. Partial-rebreathing mask

B. Nasal cannula Rationale: A nasal cannula is used when the client requires a low to medium concentration of oxygen for which precise accuracy is not essential. The Venturi mask is used primarily for clients with COPD because it can accurately provide an appropriate level of supplemental oxygen, thus avoiding the risk of suppressing the hypoxic drive. The client's respiratory status does not require a partial- or non-rebreathing mask.

A client abruptly sits up in bed, reports having difficulty breathing and has an arterial oxygen saturation of 88%. Which oxygen delivery method would give the greatest level of inspired oxygen? A. Simple mask B. Nonrebreather mask C. Face tent D. Nasal cannula

B. Nonrebreather mask Rationale: A nonrebreather mask can deliver levels of the fraction of inspired oxygen (FIO2) as high as 100%. Other modes — simple mask, face tent, and nasal cannula — deliver lower levels of FIO2.

Upon providing discharge instructions home after a tonsillectomy and adenoidectomy, which is most important? A. Provide acetaminophen for pain. B. Note any frequent swallowing. C. Allow the child an age-appropriate, quiet plan. D. Stress regular fluid consumption.

B. Note any frequent swallowing. Rationale: A complication of a tonsillectomy and adenoidectomy is bleeding. If the child is bleeding he or she must be brought to the emergency room immediately. To determine if a child is bleeding, the parents must assess for frequent swallowing. All of the other discharge instructions are appropriate, but noting any frequent swallowing is the priority.

A firefighter was trapped in a fire and is admitted to the intensive care unit for smoke inhalation. After 12 hours, the firefighter is exhibiting signs of acute respiratory distress syndrome (ARDS) and is intubated. Which other supportive measure should be initiated in this client? A. Psychological counseling B. Nutritional support C. High-protein oral diet D. Occupational therapy

B. Nutritional support Rationale: Aggressive, supportive care must be provided to compensate for the severe respiratory dysfunction. This supportive therapy almost always includes intubation and mechanical ventilation. In addition, circulatory support, adequate fluid volume, and nutritional support are important. Oral intake is contraindicated by intubation. Counseling and occupational therapy would not be priorities during the acute stage of ARDS.

What intervention can be taught to the parents of a 3-year-old child with pneumonia who is not hospitalized? A. Offer the child only cool liquids B. Offer the child her favorite warm liquid drinks C. Use a warm mist humidifier D. Call the physician for respiratory rate less than 28 breaths/min

B. Offer the child her favorite warm liquid drinks Rationale: Offering the child fluids that she likes will facilitate oral intake. Warm liquids help loosen secretions.

The nurse is preparing to perform a physical examination of a child with asthma. Which technique would the nurse be least likely to perform? A. Inspection B. Palpation C. Percussion D. Auscultation

B. Palpation Rationale: When examining the child with asthma, the nurse would inspect, auscultate, and percuss. Palpation would not be used.

A nurse is caring for a 21-year-old client whose medical history includes an alpha1-antitrypsin deficiency. This client has an increased risk of which health problem? A. Pulmonary edema B. Panacinar emphysema C. Cystic fibrosis (CF) D. Empyema

B. Panacinar emphysema Rationale: This deficiency predisposes young adult clients to rapid development of panacinar lobular emphysema, even in the absence of smoking. Alpha-antitrypsin deficiency is a genetic disorder that may affect the lungs or liver. It is a risk factor for chronic obstructive pulmonary disease. Alpha1-antitrypsin is an enzyme inhibitor that protects the lung parenchyma from injury. This deficiency does not influence the client's risk of pulmonary edema, CF, or empyema.

A group of nurses is reviewing the diagnosis of cystic fibrosis. With regard to the effect of this disease on the body, which parts of the body (besides the lungs) are most affected by this disease? A. Brain and spinal cord B. Pancreas and liver C. Heart and blood vessels D. Kidney and bladder

B. Pancreas and liver Rationale: The major organs affected are the lungs, pancreas, and liver. The brain, spinal cord, heart, blood vessels, kidney and bladder are not the most affected organs.

A client is admitted to the facility with a productive cough, night sweats, and a fever. Which action is most important in the initial care plan? A. Assessing the client's temperature every 8 hours B. Placing the client in respiratory isolation C. Monitoring the client's fluid intake and output D. Wearing gloves during all client contact

B. Placing the client in respiratory isolation Rationale: Because the client's signs and symptoms suggest a respiratory infection (possibly tuberculosis), respiratory isolation is indicated. Every 8 hours isn't frequent enough to assess the temperature of a client with a fever. Monitoring fluid intake and output may be required, but the client should first be placed in isolation. The nurse should wear gloves only for contact with mucous membranes, broken skin, blood, and other body fluids and substances.

A client with chronic obstructive pulmonary disease (COPD) reports increased shortness of breath after a prolonged episode of coughing. On assessment, the nurse notes an oxygen saturation of 84%, asymmetrical chest movement, and decreased breath sounds on the right side. Which condition should the nurse suspect and which interventions should the nurse implement based on these signs and symptoms? A. Expected response to coughing; give supplemental oxygen and encourage deep breathing exercises B. Pneumothorax; give supplemental oxygen and continue to monitor the client C. Oxygen toxicity; lower any supplemental oxygen and continue to monitor the client D. Chronic atelectasis; give supplemental oxygen and encourage deep breathing exercises

B. Pneumothorax; give supplemental oxygen and continue to monitor the client Rationale: Development of a pneumothorax, a potentially life-threatening complication of COPD, may be spontaneous or related to severe coughing or large intrathoracic pressure changes. The combination of asymmetry of chest movement, differences in breath sounds, and a decreased pulse oximetry are indications of pneumothorax. In response, the nurse should administer supplemental oxygen and continue close bedside monitoring of this client. The signs and symptoms described are not normal findings after coughing or due to chronic atelectasis (alveolar collapse). While a decrease in saturation is expected after coughing, due to irritation of airways and decreased ability to fully oxygenate, the saturation was lower than expected. Oxygen toxicity occurs when too high of a concentration of oxygen is given over a period of time, which triggers a severe inflammatory response. Because no specific duration or amount of oxygen was listed and a hallmark of this condition is substernal discomfort and progressive respiratory difficulties, this was an unlikely choice.

The nurse is examining an 8-year-old boy with tachycardia and tachypnea. The nurse anticipates which test as most helpful in determining the extent of the child's hypoxia? A. Pulmonary function test B. Pulse oximetry C. Peak expiratory flow D. Chest radiograph

B. Pulse oximetry Rationale: Pulse oximetry is a useful tool for determining the extent of hypoxia. It can be used by the nurse for continuous or intermittent monitoring. Pulmonary function testing measures respiratory flow and lung volumes and is indicated for asthma, cystic fibrosis, and chronic lung disease. Peak expiratory flow testing is used to monitor the adequacy of asthma control. Chest radiographs can show hyperinflation, atelectasis, pneumonia, foreign bodies, pleural effusion, and abnormal heart or lung size.

A client who was involved in a workplace accident sustained a penetrating wound of the chest that led to acute respiratory failure. Which goal of treatment should the care team prioritize when planning this client's care? A. Facilitation of long-term intubation B. Restoration of adequate gas exchange C. Attainment of effective coping D. Self-management of oxygen therapy

B. Restoration of adequate gas exchange Rationale: The objectives of treatment are to correct the underlying cause of respiratory failure and to restore adequate gas exchange in the lung. This is priority over coping and self-care. Long-term ventilation may or may not be indicated.

A nurse is developing a teaching plan for a client with chronic obstructive pulmonary disease. What should the nurse include as the most important area of teaching? A. Avoiding extremes of heat and cold B. Setting and accepting realistic short- and long-term goals C. Adopting a lifestyle of moderate activity D. Avoiding emotional disturbances and stressful situations

B. Setting and accepting realistic short- and long-term goals Rationale: A major area of teaching involves setting and accepting realistic short- and long-term goals. The other options should also be included in the teaching plan, but they are not areas that are as high a priority as setting and accepting realistic goals.

A client seeks medical attention for a new onset of fatigue and changes in coordination. Which additional assessment finding indicates to the nurse that the client is demonstrating signs of low oxygenation? Select all that apply. A. Cough B. Shortness of breath C. Drowsiness D. Impaired thought process E. Agitation

B. Shortness of breath D. Impaired thought process E. Agitation Rationale: A change in the client's respiratory rate or pattern may be one of the earliest indicators of the need for oxygen therapy. These changes may result from hypoxemia or hypoxia. Severe hypoxia can be life threatening. The signs and symptoms signaling the need for supplemental oxygen may depend on how suddenly this need develops. The client has new onset of symptoms so the low oxygenation will be associated with acute hypoxia. With acute hypoxia, changes occur in the central nervous system because the neurologic centers are very sensitive to oxygen deprivation. Acute hypoxia that is newly presenting may manifest in signs such as shortness of breath, impaired thought process, and agitation. With long-standing or chronic hypoxia that is not manifesting as a new onset of symptoms, the client may demonstrate apathy, drowsiness, and delayed reaction time. The client may also demonstrate symptoms similar to alcohol intoxication such as impaired judgment. The presence of cough is not a manifestation of acute or chronic hypoxia.

Which electrolyte does the client with cystic fibrosis need in abundance? A. Potassium B. Sodium C. Chlorine D. Magnesium

B. Sodium Rationale: Dietary intake of sodium is encouraged due to increased sodium losses. Clients are especially encouraged to eat salty pretzels, potato chips, etc. during hot weather or when sodium losses are anticipated.

A patient comes to the clinic for the third time in 2 months with chronic bronchitis. What clinical symptoms does the nurse anticipate assessing for this patient? A. Chest pain during respiration B. Sputum and a productive cough C. Fever, chills, and diaphoresis D. Tachypnea and tachycardia

B. Sputum and a productive cough Rationale: Chronic bronchitis, a disease of the airways, is defined as the presence of cough and sputum production for at least 3 months in each of 2 consecutive years

A nurse admits a new client with acute respiratory failure. What are the clinical findings of a client with acute respiratory failure? A. Insidious onset of lung impairment in a client who had normal lung function B. Sudden onset of lung impairment in a client who had normal lung function C. Insidious onset of lung impairment in a client who had compromised lung function D. Sudden onset of lung impairment in a client who had compromised lung function

B. Sudden onset of lung impairment in a client who had normal lung function Rationale: In acute respiratory failure, the ventilation or perfusion mechanisms in the lung are impaired. Acute respiratory failure occurs suddenly in a client who previously had normal lung function.

The nurse is caring for a child who has been admitted with a possible diagnosis of cystic fibrosis. Which laboratory/diagnostic tools would most likely be used to help determine the diagnosis of this child? A. Purified protein derivative test B. Sweat sodium chloride test C. Blood culture and sensitivity D. Pulmonary functions test

B. Sweat sodium chloride test Rationale: Sweat sodium chloride tests are used for determining the diagnosis of cystic fibrosis. Purified protein derivative tests are used to detect TB. Blood culture and sensitivity is done to determine the causative agent as well as the anti-infective needed to treat an infection. Pulmonary function tests are diagnostic tools for the child with asthma and indicate the amount of obstruction in the bronchial airways, especially in the smallest airways of the lungs.

A client is prescribed postural drainage because secretions are accumulating in the upper lobes of the lungs. The nurse instructs the client to: A. Lay in bed with the head on a pillow. B. Take prescribed albuterol (Ventolin) before performing postural drainage. C. Perform drainage 1 hour after meals. D. Hold each position for 5 minutes.

B. Take prescribed albuterol (Ventolin) before performing postural drainage. Rationale: When a client is to perform postural drainage, the nurse should instruct the client to use the prescribed bronchodilator (e.g., albuterol) first. This will open airways and promote drainage. The client is to perform postural drainage before meals, not after. This will aid in preventing nausea, vomiting, and aspiration. For secretions accumulated in the upper lobes, the client will sit up or even lean forward while sitting. Head on a pillow is not a sufficient increase in height. The client is also to lay in each position for 10 to 15 minutes.

The nurse has explained to the client that after his thoracotomy, it will be important to adhere to a coughing schedule. The client is concerned about being in too much pain to be able to cough. What would be an appropriate nursing intervention for this client? A. Teach him postural drainage. B. Teach him how to perform huffing. C. Teach him to use a mini-nebulizer. D. Teach him how to use a metered dose inhaler.

B. Teach him how to perform huffing. Rationale: The technique of "huffing" may be helpful for the client with diminished expiratory flow rates or for the client who refuses to cough because of severe pain. Huffing is the expulsion of air through an open glottis. Inhalers, nebulizers, and postural drainage are not substitutes for performing coughing exercises.

The child has been admitted to the hospital with a possible diagnosis of pneumonia. Which finding(s) is consistent with this diagnosis? Select all that apply. A. The child's temperature is 98.4° F (36.9°C). B. The child's chest x-ray indicates the presence of perihilar infiltrates. C. The child's white blood cell count is elevated. D. The child's respiratory rate is rapid. E. The child is producing yellow purulent sputum.

B. The child's chest x-ray indicates the presence of perihilar infiltrates. C. The child's white blood cell count is elevated. D. The child's respiratory rate is rapid. E. The child is producing yellow purulent sputum. Rationale: Children with pneumonia may exhibit the following: a chest x-ray with perihilar infiltrates, an elevated leukocyte level, an increased respiratory rate, and a productive cough. The child with pneumonia typically has a fever.

A nurse is caring for an infant admitted with a diagnosis of bronchiolitis. After completing an assessment, the nurse creates a plan of care for the infant. Which client goal would be priority in the plan of care? A. The infant will attain oxygen saturation of 90% on room air. B. The infant's airway will remain clear and free of mucus. C. The infant's breathing will be less labored. D. The infant will have decreased nasal stuffiness.

B. The infant's airway will remain clear and free of mucus. Rationale: Keeping the infant's airway clear is the top priority. An O2 saturation of 90% on room air is minimally acceptable. It is important that the infant's breathing be less labored and that there is decreased nasal stuffiness, but having the airway clear and free of mucus is most important.

The nurse encourages the mother of a toddler with acute LTB to stay at the bedside as much as possible. The nurses rationale for this action is primarily that: A. Mothers of hospitalized toddlers often experience guilt B. The mothers presence will reduce anxiety and ease child respiratory efforts C. Separation from mother is a major developmental threat at this age D. The mother can provide constant observations of the childs respiratory efforts

B. The mothers presence will reduce anxiety and ease child respiratory efforts Rationale: The family's presence will decrease the childs distress

A 2-year-old toddler is seen for acute laryngotracheobronchitis. What observation would lead the nurse to suspect airway occlusion? A. The toddler states being tired and wanting to sleep. B. The respiratory rate is gradually increasing. C. The cough is becoming harsher. D. The nasal discharge is increasing.

B. The respiratory rate is gradually increasing. Rationale: Acute laryngotracheobronchitis is also know as croup. It produces edema of the larynx, trachea, and bronchi. An increasing respiratory rate, retractions, and nasal flaring are signs of major respiratory distress and occlusion. The toddler is breathing faster because less air is received with each breath. Nasal discharge is generally not seen with croup. The cough of croup is due to the inflammation in the larynx and trachea and it is a barking cough (sounds like a seal). A 2-year-old toddler will become tired and fall asleep or be irritable and unable to fall asleep. This age group is unable to verbalize being tired and wanting to sleep.

The nurse is assessing a 5-year-old girl who is anxious, has a high fever, speaks in a whisper, and sits up with her neck thrust forward. Based on these findings, what would be least appropriate for the nurse to perform? A. Providing 100% oxygen B. Visualizing the throat C. Having the child sit forward D. Auscultating for lung sounds

B. Visualizing the throat Rationale: The child is exhibiting signs and symptoms of epiglottitis, which can be life- threatening. Under no circumstances should the nurse attempt to visualize the throat. Reflex laryngospasm may occur, precipitating immediate airway occlusion. Providing 100% oxygen in the least invasive manner that is most acceptable to the child is a sound intervention, as is allowing the child to assume a position of sitting forward with the neck extended. Auscultation would reveal breath sounds consistent with an obstructed airway.

Which type of ventilator has a preset volume of air to be delivered with each inspiration? A. Negative-pressure B. Volume-controlled C. Time-cycled D. Pressure-cycled

B. Volume-controlled Rationale: With volume-controlled ventilation, the volume of air to be delivered with each inspiration is preset. Negative-pressure ventilators exert a negative pressure on the external chest. Time-cycled ventilators terminate or control inspiration after a preset time. When the pressure-cycled ventilator cycles on, it delivers a flow of air (inspiration) until it reaches a preset pressure, and then cycles off, and expiration occurs passively.

A nurse's assessment reveals that a client with chronic obstructive pulmonary disease may be experiencing bronchospasm. Which assessment findings would suggest that the client is experiencing bronchospasm? Select all that apply. A. Fine or coarse crackles on auscultation B. Wheezes or diminished breath sounds on auscultation C. Reduced respiratory rate or lethargy D. Slow, deliberate respirations and diaphoresis E. Labored and rapid breathing

B. Wheezes or diminished breath sounds on auscultation E. Labored and rapid breathing Rationale: Wheezing and diminished breath sounds are consistent with bronchospasm. Crackles are usually attributable to other respiratory or cardiac pathologies. Bronchospasm usually results in rapid, labored breathing and agitation, not slow, deliberate respirations, reduced respiratory rate, or lethargy.

The nurse is caring for a child admitted with asthma. Which clinical manifestations would likely have been noted in the child with this diagnosis? A. Elevated temperature B. Wheezing C. Circumoral cyanosis D. Clubbed fingers

B. Wheezing Rationale: Symptoms of asthma include dry hacking cough, wheezing (the sound of expired air being pushed through obstructed bronchioles), and difficulty breathing. Elevated temperature is not usually seen. Circumoral cyanosis is seen with a diagnosis of pneumonia, and clubbing of the fingers is seen in cystic fibrosis.

During discharge teaching, a nurse is instructing a client about pneumonia. The client demonstrates his understanding of relapse when he states that he must: A. follow up with the physician in 2 weeks. B. continue to take antibiotics for the entire 10 days. C. turn and reposition himself every 2 hours. D. maintain fluid intake of 40 oz (1,200 ml) per day.

B. continue to take antibiotics for the entire 10 days. Rationale: The client demonstrates understanding of how to prevent relapse when he states that he must continue taking the antibiotics for the prescribed 10-day course. Although the client should keep the follow-up appointment with the physician and turn and reposition himself frequently, these interventions don't prevent relapse. The client should drink 51 to 101 oz (1,500 to 3,000 ml) per day of clear liquids.

The classification of Stage II of COPD is defined as A. at risk for COPD. B. moderate COPD. C. severe COPD. D. very severe COPD. E. mild COPD.

B. moderate COPD. Rationale: Stage II is moderate COPD. Stage 0 is at risk for COPD. Stage I is mild COPD. Stage III is severe COPD. Stage IV is very severe COPD.

Arterial blood gas analysis would reveal which value related to acute respiratory failure? A. PaO2 80 mm Hg B. pH 7.28 C. PaCO2 32 mm Hg D. pH 7.35

B. pH 7.28 Rationale: Acute respiratory failure is defined as a decrease in arterial oxygen tension (PaO2) to less than 60 mm Hg (hypoxemia) and an increase in arterial carbon dioxide tension (PaCO2) to greater than 50 mm Hg (hypercapnia), with an arterial pH less than 7.35.

A rapid strep test has confirmed that a 5-year-old girl has a group A Streptococcus infection. When teaching the parents about measures to implement, which of the following would be the least immediate concern? A) Using a cool mist humidifier B) Encouraging the child to drink liquids C) Discarding the child's toothbrush D) Administering antibiotic therapy

C) Discarding the child's toothbrush

A nurse is preparing a teaching program for a parenting group about preventing foreign body aspiration. Which of the following would the nurse include? A) Avoid giving popcorn to children younger than the age of 2 years. B) Withhold peanuts from children until they are at least 5 years of age. C) If an object fits through a standard toilet paper roll, the child can aspirate it. D) Keep pennies and dimes out of the child's reach; quarters do not pose a problem.

C) If an object fits through a standard toilet paper roll, the child can aspirate it.

A nurse is teaching a client about using an incentive spirometer. Which statement by the nurse is correct? A. "Breathe in and out quickly." B. "You need to start using the incentive spirometer 2 days after surgery." C. "Before you do the exercise, I'll give you pain medication if you need it." D. "Don't use the incentive spirometer more than 5 times every hour."

C. "Before you do the exercise, I'll give you pain medication if you need it." Rationale: The nurse should assess the client's pain level before the client does incentive spirometry exercises and administer pain medication as needed. Doing so helps the client take deeper breaths and help prevents atelectasis. The client should breathe in slowly and steadily and hold the breath for 3 seconds after inhalation. The client should start doing incentive spirometry immediately after surgery and aim to do 10 incentive spirometry breaths every hour.

A nurse has just completed teaching with a client who has been prescribed a meter-dosed inhaler for the first time. Which statement if made by the client would indicate to the nurse that further teaching and follow-up care is necessary? A. "I will make sure to take a slow, deep breath as I push on my inhaler." B. "After I breathe in, I will hold my breath for 10 seconds." C. "I do not need to rinse my mouth with this type of inhaler." D. "If I use the spacer, I know I am only supposed to push on the inhaler once."

C. "I do not need to rinse my mouth with this type of inhaler." Rationale: Mouth-washing and spitting are effective in reducing the amount of drug swallowed and absorbed systemically. Actuation during a slow (30 L/min or 3 to 5 seconds) and deep inhalation should be followed by 10 seconds of holding the breath. The client should actuate only once. Simple tubes do not obviate the spacer/VHC per inhalation.

A patient taking isoniazid (INH) therapy for tuberculosis demonstrates understanding when making which statement? A. "I am going to have a tuna fish sandwich for lunch." B. "It is all right if I drink a glass of red wine with my dinner." C. "It is all right if I have a grilled cheese sandwich with American cheese." D. "It is fine if I eat sushi with a little bit of soy sauce."

C. "It is all right if I have a grilled cheese sandwich with American cheese." Rationale: Patients taking INH should avoid foods that contain tyramine and histamine (tuna, aged cheese, red wine, soy sauce, yeast extracts), because eating them while taking INH may result in headache, flushing, hypotension, lightheadedness, palpitations, and diaphoresis. Patients should also avoid alcohol because of the high potential for hepatotoxic effects.

A parent asks the nurse about immunizing her 7-month-old daughter against the flu. Which response by the nurse would be most appropriate? A. "She really doesn't need the vaccine until she reaches 1 year of age." B. "She will probably receive it the next time she is to get her routine shots." C. "Since your daughter is older than 6 months, she should get the vaccine every year." D. "The vaccine has many side effects, so she wouldn't get it until she's ready to go to school."

C. "Since your daughter is older than 6 months, she should get the vaccine every year." Rationale: The current recommendations are for all children older than 6 months of age to be immunized yearly against influenza.

A client has just been diagnosed with lung cancer. After the health care provider discusses treatment options and leaves the room, the client asks the nurse how the treatment is decided upon. What would be the nurse's best response? A. "The type of treatment depends on the client's age and health status." B. "The type of treatment depends on what the client wants when given the options." C. "The type of treatment depends on the cell type of the cancer, the stage of the cancer, and the client's health status." D. "The type of treatment depends on the discussion between the client and the health care provider of which treatment is best."

C. "The type of treatment depends on the cell type of the cancer, the stage of the cancer, and the client's health status." Rationale: Treatment of lung cancer depends on the cell type, the stage of the disease, and the client's physiologic status (particularly cardiac and pulmonary status). Treatment does not depend solely on the client's age or the client's preference between the different treatment modes. The decision about treatment does not primarily depend on a discussion between the client and the health care provider of which treatment is best, though this discussion will take place.

A client with chronic obstructive pulmonary disease (COPD) and cor pulmonale is being prepared for discharge. The nurse should provide which instruction? A. "Limit yourself to smoking only 2 cigarettes per day." B. "Eat a high-sodium diet." C. "Weigh yourself daily and report a gain of 2 lb in 1 day." D. "Maintain bed rest."

C. "Weigh yourself daily and report a gain of 2 lb in 1 day." Rationale: The nurse should instruct the client to weigh himself daily and report a gain of 2 lb in 1 day. COPD causes pulmonary hypertension, leading to right-sided heart failure or cor pulmonale. The resultant venous congestion causes dependent edema. A weight gain may further stress the respiratory system and worsen the client's condition. The nurse should also instruct the client to eat a low-sodium diet to avoid fluid retention and engage in moderate exercise to avoid muscle atrophy. The client shouldn't smoke at all.

The pediatric unit has multiple clients experiencing upper respiratory system complications. Which pediatric client is at the highest risk for respiratory distress? A. 3-year-old child with croup B. 11-month-old infant with nasopharyngitis C. 2-year-old child with epiglottitis D. 16-year-old adolescent with asthma

C. 2-year-old child with epiglottitis Rationale: Epiglottitis is a medical emergency due to the swelling of the epiglottis covering the larynx. This client needs frequent assessment for respiratory distress, especially since young children have smaller, more compliant airways. The 3-year-old child has more developed respiratory passages than a 2-year-old child, and although croup may cause respiratory distress, the likelihood of airway obstruction is lower when compared to epiglottitis. The 11-month-old infant has a common cold, typically from a virus. The 16-year-old adolescent with asthma has fully developed respiratory airways, which are less likely to be obstructed.

The nurse working at the child community clinic must administer the influenza vaccine to the high-risk children first. Which child would she choose first? A. 12-month-old client who is very healthy B. 21-month-old client who has a cold C. 23-month-old client who had heart surgery as an infant for a defect D. 22-month-old client who has a wound from touching a hot pan at home

C. 23-month-old client who had heart surgery as an infant for a defect Rationale: Children who are considered high risk and could benefit from the influenza vaccine are: immunocompromise, have a chronic pulmonary disease, have had a congenital abnormality, chronic renal or metabolic disease, sickle-cell disease, HIV, any type of neurological disorder (seizures). The other choices would be considered normal and the child is not at high risk.

The nurse is assessing several children. Which child is most at risk for dysphagia? A. 7-month-old with erythematous rash B. 8-year-old with fever and fatigue C. 5-year-old with epiglottitis D. 2-month-old with toxic appearance

C. 5-year-old with epiglottitis Rationale: The 5-year-old with epiglottitis has a sore, swollen throat placing the child at risk for dysphagia (difficulty swallowing). Erythematous rash and mild toxic appearance are typical of influenza. Fever and fatigue are symptoms of a common cold. Influenza and the common cold may cause sore throats but would not be the highest risk for dysphagia.

A client with a respiratory condition is receiving oxygen therapy. While assessing the client's PaO2, the nurse knows that the therapy has been effective based on which of the following readings? A. 45 mm Hg B. 58 mm Hg C. 84 mm Hg D. 120 mm Hg

C. 84 mm Hg Rationale: In general, clients with respiratory conditions are given oxygen therapy only to increase the arterial oxygen pressure (PaO2) back to the client's normal baseline, which may vary from 60 to 95 mm Hg.

The nurse is caring for a patient with COPD. The patient is receiving oxygen therapy via nasal cannula. The nurse understands that the goal of oxygen therapy is to maintain the patient's SaO2 level at or above what percent? A. 50% B. 70% C. 90% D. 30%

C. 90% Rationale: The goal of supplemental oxygen therapy is to increase the baseline resting partial arterial pressure of oxygen (PaO2) to at least 60 mm Hg at sea level and arterial oxygen saturation (SaO2) to at least 90%.

The nurse is assessing a client whose respiratory disease is characterized by chronic hyperinflation of the lungs. Which physical characteristic would the nurse most likely observe in this client? A. Signs of oxygen toxicity B. A moon face C. A barrel chest D. Long, thin fingers

C. A barrel chest Rationale: In chronic obstructive pulmonary disease (COPD) clients with a primary emphysematous component, chronic hyperinflation leads to the barrel chest thorax configuration. The client with COPD is more likely to have finger clubbing, which is an abnormal rounded appearance of the fingertips, rather than long, thin fingers. Clubbed fingers are the result of chronically low blood levels of oxygen. A moon face is swelling of the face due to increased fat deposits. This may be a sign of Cushing syndrome or a side effect of steroid use. Signs of oxygen toxicity, such as facial pallor or behavioral changes, may be possible but are not the most likely physical findings for this client.

A nurse is caring for a group of clients on a medical-surgical floor. Which client is at greatest risk for developing pneumonia? A. A client with a history of smoking two packs of cigarettes per day until quitting 2 years ago B. A client who ambulates in the hallway every 4 hours C. A client with a nasogastric tube D. A client who is receiving acetaminophen (Tylenol) for pain

C. A client with a nasogastric tube Rationale: Nasogastric, orogastric, and endotracheal tubes increase the risk of pneumonia because of the risk of aspiration from improperly placed tubes. Frequent oral hygiene and checking tube placement help prevent aspiration and pneumonia. Although a client who smokes is at increased risk for pneumonia, the risk decreases if the client has stopped smoking. Ambulation helps prevent pneumonia. A client who receives opioids, not acetaminophen, has a risk of developing pneumonia because respiratory depression may occur.

A client presents to the emergency department after being in a boating accident about 3 hours ago. Now the client reports headache, fatigue, and the feeling of not being able to breathe enough. The nurse notes that the client is restless and tachycardic with an elevated blood pressure. This client may be in the early stages of which respiratory problem? A. Pneumoconiosis B. Pleural effusion C. Acute respiratory failure D. Pneumonia

C. Acute respiratory failure Rationale: Early signs of acute respiratory failure are those associated with impaired oxygenation and may include restlessness, fatigue, headache, dyspnea, air hunger, tachycardia, and increased blood pressure. As the hypoxemia progresses, more obvious signs may be present, including confusion, lethargy, tachycardia, tachypnea, central cyanosis, diaphoresis, and, finally, respiratory arrest. Pneumonia is infectious and would not result from trauma. Pneumoconiosis results from exposure to occupational toxins. A pleural effusion does not cause this constellation of symptoms.

A school nurse is caring for a 10-year-old client who is having an asthma attack. What is the preferred intervention to alleviate this client's airflow obstruction? A. Administer corticosteroids by metered dose inhaler. B. Administer inhaled anticholinergics. C. Administer an inhaled beta-adrenergic agonist. D. Use a peak flow monitoring device.

C. Administer an inhaled beta-adrenergic agonist. Rationale: Asthma exacerbations are best managed by early treatment and education of the client. Quick-acting beta-adrenergic medications are the first used for prompt relief of airflow obstruction. Systemic corticosteroids may be necessary to decrease airway inflammation in clients who fail to respond to inhaled beta-adrenergic medication. A peak flow device will not resolve short-term shortness of breath.

Which action should the nurse take first when providing care for a client during an acute asthma attack? A. Obtain arterial blood gases. B. Send for STAT chest x-ray. C. Administer prescribed short-acting bronchodilator. D. Initiate oxygen therapy and reassess pulse oximetry in 10 minutes.

C. Administer prescribed short-acting bronchodilator. Rationale: Administering a short-acting bronchodilator will dilate the airway and enable oxygen to reach the lungs. Although ABGs and a chest x-ray are valid diagnostic tests for lung disorders, immediate action to restore gas exchange is a priority in an acute asthma attack. The administration of oxygen is indicated, but without open bronchioles, the action will not be effective in an acute attack.

When assessing for substances that are known to harm workers' lungs, the occupational health nurse should assess their potential exposure to which of the following? A. Organic acids B. Solvents C. Asbestos D. Gypsum

C. Asbestos Rationale: Pneumoconiosis is a general term given to any lung disease caused by dusts that are breathed in and then deposited deep in the lungs causing damage. Pneumoconiosis is usually considered an occupational lung disease, and includes asbestosis, silicosis, and coal workers' pneumoconiosis, also known as "Black Lung Disease." Asbestos is among the more common causes of pneumoconiosis. Organic acids, solvents, and gypsum do not have this effect.

A nurse is completing a focused respiratory assessment of a child with asthma. What assessment finding is most closely associated with the characteristic signs and symptoms of asthma? A. Shallow respirations B. Increased anterior-posterior (AP) diameter C. Bilateral wheezes D. Bradypnea

C. Bilateral wheezes Rationale: The three most common symptoms of asthma are cough, dyspnea, and wheezing. There may be generalized wheezing (the sound of airflow through narrowed airways), first on expiration and then, possibly, during inspiration as well. Respirations are not usually slow and the child's AP diameter does not normally change.

The clinic nurse is caring for a client with acute bronchitis. The client asks what may have caused the infection. What may induce acute bronchitis? A. Aspiration B. Drug ingestion C. Chemical irritation D. Direct lung damage

C. Chemical irritation Rationale: Chemical irritation from noxious fumes, gases, and air contaminants induces acute bronchitis. Aspiration related to near drowning or vomiting, drug ingestion or overdose, and direct damage to the lungs are factors associated with the development of acute respiratory distress syndrome.

The nurse notices that a child is spitting up small amounts of blood in the immediate postoperative period after a tonsillectomy. What would be the best intervention? A. Suction the back of the throat. B. Encourage the child to cough. C. Continue to assess for bleeding. D. Notify the health care immediately.

C. Continue to assess for bleeding. Rationale: Children will have a small amount of blood mixed with saliva following a tonsillectomy. Suctioning or coughing could irritate the surgical site and cause hemorrhage.

The nurse is preparing to discharge a client after thoracotomy. The client is going home on oxygen therapy and requires wound care. As a result, the client will receive home care nursing. Which information should the nurse include in discharge teaching for this client? A. Safe technique for self-suctioning of secretions B. Technique for performing postural drainage C. Correct and safe use of oxygen therapy equipment D. How to provide safe and effective tracheostomy care

C. Correct and safe use of oxygen therapy equipment Rationale: Respiratory care and other treatment modalities (oxygen, incentive spirometry, chest physiotherapy [CPT], and oral, inhaled, or intravenous medications) may be continued at home. Therefore, the nurse needs to instruct the client and family in their correct and safe use. The scenario does not indicate the client needs suctioning, postural drainage, or tracheostomy care.

A client has asthma. Which of the following medications is a commonly prescribed mast cell stabilizer used for asthma? A. Albuterol B. Budesonide C. Cromolyn sodium D. Theophylline

C. Cromolyn sodium Rationale: Cromolyn sodium and nedocromil are mild to moderate anti-inflammatory agents and are considered alternative medications for treatment. These medications stabilize mast cells. Though also used in the treatment of asthma the following are NOT mast cell stabilizers: Albuterol is a beta2-antagonist. Budesonide is an inhaled corticosteroid. Theophylline is a mild to moderate bronchodilator.

When performing the physical examination of a child with cystic fibrosis, what would the nurse expect to assess? A. Dullness over the lung fields B. Increased diaphragmatic excursion C. Decreased tactile fremitus D. Hyperresonance over the liver

C. Decreased tactile fremitus Rationale: Examination of a child with cystic fibrosis typically reveals decreased tactile fremitus over areas of atelectasis, hyperresonance over the lung fields from air trapping, decreased diaphragmatic excursion, and dullness over the liver when enlarged.

A nurse is preparing to care for a client with bronchiectasis. The nurse should recognize that this client is likely to experience respiratory difficulties related to what pathophysiologic process? A. Intermittent episodes of acute bronchospasm B. Alveolar distention and impaired diffusion C. Dilation of bronchi and bronchioles D. Excessive gas exchange in the bronchioles

C. Dilation of bronchi and bronchioles Rationale: Bronchiectasis is a chronic, irreversible dilation of the bronchi and bronchioles that results from destruction of muscles and elastic connective tissue. It is not characterized by acute bronchospasm, alveolar distention, or excessive gas exchange.

Teaching safety precautions with the administration of antihistamines is important because of what common side effect? A. Dry mouth B. Excitability C. Drowsiness D. Dry mucous membranes

C. Drowsiness Rationale: Drowsiness is a safety hazard when alertness is needed, especially with a teenage driver. Nonsedating brands should be used.

The nurse in the intensive care unit is caring for a client with pulmonary hypertension. Which finding should the nurse expect to assess? A. Pulmonary artery pressure greater than 20 mm Hg B. Flat neck veins C. Dyspnea at rest D. Enlarged spleen

C. Dyspnea at rest Rationale: The main symptom in pulmonary hypertension is dyspnea. At first dyspnea occurs with exertion, then eventually at rest. A client with pulmonary hypertension will have a pulmonary artery pressure greater than 25 mm Hg at rest and distended neck veins secondary to right-sided heart failure. The nurse would expect the liver, not the spleen, to be enlarged secondary to engorgement in pulmonary hypertension.

Once an allergen is identified in a child with allergic rhinitis, the treatment of choice about which to educate the parents is: A. Using the appropriate medications B. Beginning desensitization injections C. Eliminating the allergen D. Removing the adenoids

C. Eliminating the allergen Rationale: The first priority is to attempt to remove the causative agent from the childs environment.

An 8-year-old with cystic fibrosis has had a noted decline on the growth chart. Which nursing intervention is best for maintaining adequate nutrition? A. Provide high caloric meals to the client's liking. B. Delay pancreatic enzymes until food enters the small intestine. C. Encourage high calorie, high protein snacks. D. Limit sodium to a 2 gram sodium restricted diet

C. Encourage high calorie, high protein snacks. Rationale: The best nursing intervention is a high calorie, high protein snack. Calories can be obtained from non-nutritious foods. It is not only that the client needs calories for energy, but nutrition needs to be present. Pancreatic enzymes aid in digestion so they need to be available for foods; thus they are given prior to ingestion. Sodium is encouraged due to the high sodium loss.

You are caring for a client who has been diagnosed with viral pneumonia. You are making a plan of care for this client. What nursing interventions would you put into the plan of care for a client with pneumonia? A. Give antibiotics as ordered. B. Place client on bed rest. C. Encourage increased fluid intake. D. Offer nutritious snacks 2 times a day.

C. Encourage increased fluid intake. Rationale: The nurse places the client in semi-Fowler's position to aid breathing and increase the amount of air taken with each breath. Increased fluid intake is important to encourage because it helps to loosen secretions and replace fluids lost through fever and increased respiratory rate. The nurse monitors fluid intake and output, skin turgor, vital signs, and serum electrolytes. He or she administers antipyretics as indicated and ordered. Antibiotics are not given for viral pneumonia. The client's activity level is ordered by the physician, not decided by the nurse.

Which statement, if made by parents of a child with CF, indicates they understood the nurse teaching on pancreatic enzyme replacement? A. Enzymes will improve my childs breathing B. I should give the enzymes 1 hour after meals C. Enzymes should be given with meals and snacks D. The enzymes are stopped if my child begins wheezing

C. Enzymes should be given with meals and snacks Rationale: Children with CF need to take enzymes with food for adequate absorption of nutrients.

What is a common trigger for asthma attacks in children? A. Febrile episodes B. Dehydration C. Exercise D. Seizures

C. Exercise Rationale: Exercise is one of the most common triggers for asthma attacks, particularly in school-age children.

The nurse is caring for a child immediately following a tonsillectomy. The child requests something to drink. Which action by the nurse is best? A. Inform the child he or she can have nothing to drink for a few hours. B. Provide the child with a red popsicle to eat. C. Give the child a few ice chips to consume. D. Assess the child's gag reflex before giving oral fluids.

C. Give the child a few ice chips to consume. Rationale: Ice chips are soothing and appropriate for the child at this time. The child should not consume anything red to limit confusion between red coloring and blood. Otherwise, a popsicle would be allowed. The child does not have to wait hours following the procedure to drink. Once the child is awake, ice chips may be offered and the diet increased as tolerated, based on the prescription. The nurse would not assess the gag reflex; nothing should be placed in the child's mouth/throat as this would increase the risk of hemorrhage and infection.

A nurse is administering 100% oxygen to a child with a pneumothorax based on the understanding that this treatment is used primarily for which reason? A. Improve gas exchange B. Bypass the obstruction C. Hasten air reabsorption D. Prevent hypoxemia

C. Hasten air reabsorption Rationale: Administration of 100% oxygen is used to treat pneumothorax primarily because it hastens the reabsorption of air. Generally this is used only for a few hours. Although the oxygen also improves gas exchange and prevents hypoxemia, these are not the reasons for its use in this situation. There is no obstruction with a pneumothorax.

For which problem should the child with chronic otitis media with effusion be evaluated? A. Brain abscess B. Meningitis C. Hearing loss D. Perforation of tympanic membrane

C. Hearing loss Rationale: Chronic otitis media with effusion is the most common cause of hearing loss in children.

Which immunizations are suggested at 2 months of age to prevent bacterial pneumonia? Select all that apply. A. Rubella B. Rotavirus C. Hib D. Varicella E. Pneumococcal

C. Hib E. Pneumococcal Rationale: The Hib (H. influenzae type B conjugate) and pneumococcal (PCV) vaccines are suggested at 2 months of age and were developed to decrease the incidence of respiratory infections, including bacterial pneumonia. The rubella vaccine protects against the German measles. The rotavirus vaccine protects against dehydration from a gastrointestinal virus. The varicella vaccine protects against the chickenpox virus.

Which statement made by a parent indicates an understanding about treatment of streptococcal pharyngitis? A. I guess my child will need to have his tonsils removed B. A couple of days of rest and some ibuprofen will take care of this C. I should give the penicillin three times a day for 10 days D. I am giving my child prednisone to decrease the swelling of the tonsils

C. I should give the penicillin three times a day for 10 days Rationale: Streptococcal pharyngitis is best treated with oral penicillin two to three times daily for 10 days.

A nurse is developing a care plan for a client with chronic obstructive pulmonary disease (COPD) admitted to the hospital for the second time this year with pneumonia. Which nursing diagnoses would be appropriate for this client? Select all that apply. A. Ineffective airway clearance related to inhalation of toxins B. Activity intolerance related to oxygen supply and demand C. Impaired gas exchange related to ventilation-perfusion inequality D. Ineffective health management related to fatigue E. Deficient knowledge regarding self-care related to preventable complications

C. Impaired gas exchange related to ventilation-perfusion inequality E. Deficient knowledge regarding self-care related to preventable complications Rationale: Impaired gas exchange and deficient knowledge are the appropriate diagnoses for this client based on the information provided. Pneumonia is an acute infection of the parenchyma whose pathophysiology typically triggers an inflammatory response in the lung. In a client with COPD who already has chronic inflammation, gas exchange becomes further compromised. Areas of the lung receive either oxygen but no blood flow or blood flow but no oxygen (ventilation/perfusion inequality). Because this was the second admission for the same diagnosis, deficient knowledge of prevention strategies should be included for this client. Although ineffective airway clearance is a possibility, not enough information is provided to conclude that it was a result of toxins such as cigarette smoke. Activity intolerance and health management should be addressed as a risk because pneumonia and COPD impact activity and cause fatigue, but not enough information was provided to make these a problem.

The nurse is discussing discharge instructions with the parents of a 6-year-old who had a tonsillectomy. What is the most important thing to stress? A. Administer analgesics. B. Encourage the child to drink liquids. C. Inspect the throat for bleeding. D. Apply an ice collar.

C. Inspect the throat for bleeding. Rationale: Inspecting the throat for bleeding is the most important discharge information to give the parents. Hemorrhage is unusual postoperatively but may occur any time from the immediate postoperative period to as late as 10 days after surgery. The nurse should inspect the throat for bleeding. Mucus tinged with blood may be expected, but fresh blood in the secretions indicates bleeding. Administering analgesics, encouraging fluids and applying an ice color are important but not as important as assessing for bleeding.

A client with chronic obstructive pulmonary disease (COPD) is intubated and placed on continuous mechanical ventilation. Which equipment is most important for the nurse to keep at this client's bedside? A. Tracheostomy cleaning kit B. Water-seal chest drainage set-up C. Manual resuscitation bag D. Oxygen analyzer

C. Manual resuscitation bag Rationale: The client with COPD depends on mechanical ventilation for adequate tissue oxygenation. The nurse must keep a manual resuscitation bag at the bedside to ventilate and oxygenate the client in case the mechanical ventilator malfunctions. Because the client doesn't have chest tubes or a tracheostomy, keeping a water-seal chest drainage set-up or a tracheostomy cleaning kit at the bedside isn't necessary. Although the nurse may keep an oxygen analyzer (pulse oximeter) on hand to evaluate the effectiveness of ventilation, this equipment is less important than the manual resuscitation bag.

What is histamine, a mediator that supports the inflammatory process in asthma, secreted by? A. Eosinophils B. Lymphocytes C. Mast cells D. Neutrophils

C. Mast cells Rationale: Mast cells, neutrophils, eosinophils, and lymphocytes play key roles in the inflammation associated with asthma. When activated, mast cells release several chemicals called mediators. One of these chemicals is called histamine.

Which clinical manifestation of acute nasopharyngitis is more of a concern for the infant than the older child? A. Fever B. Vomiting C. Nasal congestion D. Diarrhea

C. Nasal congestion Rationale: The infant has smaller airways, making it more difficult to breathe when nasal congestion occurs. The older child can tolerate the congestion better than the infant with smaller airways. Depending upon the age of the child, younger infants are afebrile. Vomiting and diarrhea can occur at any age as the mucus from the nasal drainage enters the gastrointestinal tract.

A nurse is teaching a client with asthma about the proper use of the prescribed inhaled corticosteroid. Which adverse effect should the nurse be sure to address in client teaching? A. Increased respiratory secretions B. Bradycardia C. Oral candidiasis D. Decreased level of consciousness

C. Oral candidiasis Rationale: Thrush or oral candidiasis is a fungal infection that presents with white lesions on the tongue and/or inner cheeks of the mouth. Clients should rinse their mouth after administration or use a spacer to prevent thrush, a common complication associated with use of inhaled corticosteroids. Increased respiratory secretions normally do not occur, although a cough may develop. Tachycardia, or a fast heart rate, rather than bradycardia, or a slow heart rate, is listed as an adverse effect. A decreased level of consciousness is not associated with this medication because it does not cause sedation nor is it an opiate.

The caregivers of a child who was diagnosed with cystic fibrosis 5 months ago report that they have been following all of the suggested guidelines for nutrition, fluid intake, and exercise, but the child has been having bouts of constipation and diarrhea. The nurse tells the caregiver to increase the amount of which substance in the child's diet? A. Iodized salt B. Saturated fat C. Pancreatic enzymes D. Calories from protein

C. Pancreatic enzymes Rationale: Adequate nutrition helps the child resist infections. Pancreatic enzymes must be administered with all meals and snacks. If the child has bouts of diarrhea or constipation, the dosage of enzymes may need to be adjusted. The child's diet should be high in carbohydrates and protein with no restriction of fats. The child may need 1.5 to 2 times the normal caloric intake to promote growth. Low-fat products can be selected if desired. The child also may require additional salt in the diet. Increased caloric intake compensates for impaired absorption.

The nurse is caring for a client with bronchiectasis. Chest auscultation reveals the presence of copious secretions. What intervention should the nurse prioritize in this client's care? A. Oral administration of diuretics B. Intravenous fluids to reduce the viscosity of secretions C. Postural chest drainage D. Pulmonary function testing

C. Postural chest drainage Rationale: Postural drainage is part of all treatment plans for bronchiectasis, because draining of the bronchiectatic areas by gravity reduces the amount of secretions and the degree of infection. Diuretics and IV fluids will not aid in the mobilization of secretions. Lung function testing may be indicated, but this assessment will not relieve the client's symptoms.

Which should a nurse encourage in clients who are at the risk of pneumococcal and influenza infections? A. Mobilizing early B. Using incentive spirometry C. Receiving vaccinations D. Using prescribed opioids

C. Receiving vaccinations Rationale: Identifying clients who are at risk for pneumonia provides a means to practice preventive nursing care. The nurse encourages clients at risk of pneumococcal and influenza infections to receive vaccinations against these infections. The nurse should encourage early mobilization as indicated through agency protocol, administer prescribed opioids and sedatives as indicated, and teach or reinforce appropriate technique for incentive spirometry to prevent atelectasis.

As status asthmaticus worsens, the nurse would expect which acid-base imbalance? A. Respiratory alkalosis B. Metabolic alkalosis C. Respiratory acidosis D. Metabolic acidosis

C. Respiratory acidosis Rationale: As status asthmaticus worsens, the PaCO2 increases and the pH decreases, reflecting respiratory acidosis.

The nurse is caring for an adult client recently diagnosed with the early stages of lung cancer. The nurse is aware that the preferred method of treating clients with non-small cell tumors is what method? A. Chemotherapy B. Radiation C. Surgical resection D. Bronchoscopic opening of the airway

C. Surgical resection Rationale: Surgical resection is the preferred method of treating clients with localized non-small cell tumors with no evidence of metastatic spread and adequate cardiopulmonary function. The other listed treatment options may be considered, but surgery is preferred.

Bacterial pneumonia is suspected in a 4-year-old boy with fever, headache, and chest pain. Which assessment finding would most likely indicate the need for this child to be hospitalized? A. Fever B. Oxygen saturation level of 96% C. Tachypnea with retractions D. Pale skin color

C. Tachypnea with retractions Rationale: Pneumonia is usually a self-limiting disease. Children with bacterial pneumonia can be successfully managed at home if the work of breathing is not severe and oxygen saturation is within normal limits. Hospitalization would most likely be required for the child with tachypnea, significant retractions, poor oral intake, or lethargy for the administration of supplemental oxygen, intravenous hydration, and antibiotics. Fever, although common in children with pneumonia, would not necessitate hospitalization. An oxygen saturation level of 96% would be within normal limits. Pallor (pale skin color) occurs as a result of peripheral vasoconstriction in an effort to conserve oxygen for vital functions; this finding also would not necessitate hospitalization.

A nurse is developing a teaching plan for a client with asthma. Which teaching point has the highest priority? A. Avoid contact with fur-bearing animals. B. Change filters on heating and air conditioning units frequently. C. Take ordered medications as scheduled. D. Avoid goose down pillows.

C. Take ordered medications as scheduled. Rationale: Although avoiding contact with fur-bearing animals, changing filters on heating and air conditioning units frequently, and avoiding goose down pillows are all appropriate measures for clients with asthma, taking ordered medications on time is the most important measure in preventing asthma attacks.

A nurse is developing a teaching plan for an adult client with asthma. Which teaching point should have the highest priority in the plan of care that the nurse is developing? A. Gradually increase levels of physical exertion. B. Change filters on heaters and air conditioners frequently. C. Take prescribed medications as scheduled. D. Avoid goose-down pillows.

C. Take prescribed medications as scheduled. Rationale: Although all of the measures are appropriate for a client with asthma, taking prescribed medications on time is the most important measure in preventing asthma attacks.

Which intervention for treating croup at home should be taught to parents? A. Have a decongestant available to give the child when an attack occurs B. Have the child sleep in a dry room C. Take the child outside D. Give the child an antibiotic at bedtime

C. Take the child outside Rationale: Taking the child into the cool, humid, night air may relieve mucosal swelling and improve symptoms.

The nurse is caring for a newly admitted 3-year-old child who has been diagnosed with tuberculosis. When reviewing the child's records which finding(s) is consistent with this disease? Select all that apply. A. The child currently lives at home with parents and one sibling. B. The child has been experiencing a sore throat for the past few weeks. C. The child has been experiencing night sweats. D. The child and the family were homeless for a period of time in the past 3 months. E. The child has had recent weight loss.

C. The child has been experiencing night sweats. D. The child and the family were homeless for a period of time in the past 3 months. E. The child has had recent weight loss. Rationale: Tuberculosis is a highly contagious respiratory infection. A child who has been living in crowded locations, who is impoverished, or homeless is at an increased risk. Signs and symptoms of the disease include weight loss, night sweats, anorexia and pain. A child living in a household with parents and one sibling does not have an increased risk for infection. A sore throat is not associated with tuberculosis.

Which child requires a Mantoux test? A. The child who has episodes of nighttime wheezing and coughing B. The child who has a history of allergic rhinitis C. The child whose baby-sitter has received a tuberculosis diagnosis D. The premature infant who is being treated for apnea of infancy

C. The child whose baby-sitter has received a tuberculosis diagnosis Rationale: The Mantoux test is the initial screening mechanism for patients exposed to tuberculosis.

The home care nurse is assessing a client who requires home oxygen therapy. What criterion indicates that an oxygen concentrator will best meet the needs of the client in the home environment? A. The client desires a low-maintenance oxygen delivery system that delivers oxygen flow rates up to 6 L/min. B. The client requires a high-flow system for use with a tracheostomy collar. C. The client desires a portable oxygen delivery system that can deliver 2 L/min. D. The client's respiratory status requires a system that provides an FiO2 of 65%.

C. The client desires a portable oxygen delivery system that can deliver 2 L/min. Rationale: The use of oxygen concentrators is another means of providing varying amounts of oxygen, especially in the home setting. They can deliver oxygen flows from 1 to 10 L/min and provide an FiO2 of about 40%. They require regular maintenance and are not used for high-flow applications. The client desiring a portable oxygen delivery system of 2 L/min will benefit from the use of an oxygen concentrator.

A client presents to the walk-in clinic reporting a dry, irritating cough and production of a small amount of mucus-like sputum. The client also reports soreness in the chest in the sternal area. The nurse should suspect that the primary care provider will assess the client for which health problem? A. Pleural effusion B. Pulmonary embolism C. Tracheobronchitis D. Tuberculosis

C. Tracheobronchitis Rationale: Initially, the client with tracheobronchitis has a dry, irritating cough and expectorates a scant amount of mucoid sputum. The client may report sternal soreness from coughing and have fever or chills, night sweats, headache, and general malaise. Pleural effusion and pulmonary embolism do not normally cause sputum production and would likely cause acute shortness of breath. Hemoptysis is characteristic of tuberculosis.

A nurse is providing discharge teaching for a client with COPD. What should the nurse teach the client about breathing exercises? A. Lie supine to facilitate air entry. B. Avoid pursed-lip breathing unless absolutely necessary. C. Use diaphragmatic breathing. D. Use chest breathing.

C. Use diaphragmatic breathing. Rationale: Inspiratory muscle training and breathing retraining may help improve breathing patterns in clients with COPD. Training in diaphragmatic breathing reduces the respiratory rate, increases alveolar ventilation, and helps expel as much air as possible during expiration. Pursed-lip breathing helps slow expiration, prevents collapse of small airways, and controls the rate and depth of respiration. Diaphragmatic breathing, not chest breathing, increases lung expansion. Supine positioning does not aid breathing.

What should the nurse teach a child about using an albuterol metered-dose inhaler for exercise-induced asthma? A. Take two puffs every 6 hours around the clock B. Use the inhaler only when the child is short of breath C. Use the inhaler 30 minutes before exercise D. Take on to two puffs every morning upon waking

C. Use the inhaler 30 minutes before exercise Rationale: The appropriate time to use an inhaled beta2-agonist or cromolyn is before an event that could trigger an attack.

For a client with advanced chronic obstructive pulmonary disease (COPD), which nursing action best promotes adequate gas exchange? A. Encouraging the client to drink three glasses of fluid daily B. Keeping the client in semi-Fowler's position C. Using a Venturi mask to deliver oxygen as ordered D. Administering a sedative as ordered

C. Using a Venturi mask to deliver oxygen as ordered Rationale: The client with COPD retains carbon dioxide, which inhibits stimulation of breathing by the medullary center in the brain. As a result, low oxygen levels in the blood stimulate respiration, and administering unspecified, unmonitored amounts of oxygen may depress ventilation. To promote adequate gas exchange, the nurse should use a Venturi mask to deliver a specified, controlled amount of oxygen consistently and accurately. Drinking three glasses of fluid daily wouldn't affect gas exchange or be sufficient to liquefy secretions, which are common in COPD. Clients with COPD and respiratory distress should be placed in high Fowler's position and shouldn't receive sedatives or other drugs that may further depress the respiratory center.

The nurse at a camp for children with asthma is teaching these children about the medications they are taking and how to properly take them. The nurse recognizes that many medications used on a daily basis for the treatment of asthma are given by which method? A. Directly into the vein B. Through a gastrostomy tube C. Using a nebulizer D. Sprinkled onto the food

C. Using a nebulizer Rationale: Many of these drugs used in the treatment of asthma can be given either by a nebulizer (tube attached to a wall unit or cylinder that delivers moist air via a face mask) or a metered-dose inhaler ([MDI], which is a hand-held plastic device that delivers a premeasured dose). Emergency medications are given intravenously. Most children do not have a gastrostomy tube, and medications sprinkled on foods are given with cystic fibrosis.

A client with a severe exacerbation of chronic obstructive pulmonary disease requires reliable and precise oxygen delivery. Which mask will the nurse expect the health care provider to prescribe? A. Nonrebreathing mask B. Tracheostomy collar C. Venturi mask D. Face tent

C. Venturi mask Rationale: The Venturi mask is the most reliable and accurate method for delivering precise concentrations of oxygen through noninvasive means. It is used primarily for clients with COPD because it can accurately provide appropriate levels of supplemental oxygen, thus avoiding the risk of suppressing the hypoxic drive. The Venturi mask uses the Bernoulli principle of air entrainment (trapping the air like a vacuum), which provides a high airflow with controlled oxygen enrichment. For each liter of oxygen that passes through a jet orifice, a fixed proportion of room air is entrained. Varying the size of the jet orifice and adjusting the flow of oxygen can deliver a precise volume of oxygen. The other methods of oxygen delivery listed, the nonrebreathing mask, tracheostomy collar, and face tent, do not use the Bernoulli principle and thus lack the precision of a Venturi mask.

The health care provider has prescribed beclomethasone for long term control of asthma. The nurse is most correct to advise the client that beclomethasone is a: A. mast cell stabilizer. B. xanthine derivative. C. corticosteroid. D. leukotriene inhibitor.

C. corticosteroid. Rationale: Beclomethasone is a corticosteroid prescribed for long-term asthma control. Mast cell stabilizers help to decrease bronchospasm and mucous membrane inflammation. A xanthine derivative such as theophylline is a time-released bronchodilator. Leukotriene inhibitors help with bronchodilation and decrease airway edema.

Which acute respiratory condition is the most common in early childhood? A. asthma B. bronchiolitis C. croup D. pneumonia

C. croup Rationale: Croup is the most common acute respiratory condition in early childhood (3 months to 3 years). The cardinal sign is a "barking cough." Croup is an upper airway obstruction caused by some type of inflammation. Asthma may be caused by some allergen trigger and causes bronchoconstriction and inflammation of the lower airways. Pneumonia and bronchiolitis are infectious disorders caused by bacteria or viruses; they affect lower airways.

A 6-year-old child was diagnosed as having streptococcal pharyngitis. At the follow-up visit, the nurse will assess the child for which potential complication? A. swollen lymph nodes that obstruct the airway B. infection that may cause a tooth abscess C. development of rheumatic fever D. nephrosis of the kidney

C. development of rheumatic fever Rationale: The Group A strain of streptococci causing streptococcal pharyngitis can cause a hypersensitivity reaction that results in either rheumatic fever or glomerulonephritis. Swollen lymph nodes obstructing the airway would occur during the illness, not afterward. They would have been addressed at an emergency visit, rather than at the routine follow-up visit. The organism will not affect the teeth. Nephrosis or nephrotic syndrome relates to increased edema and protein, not the infection of the kidney (glomerulonephritis).

The nurse is collecting data on a child with a diagnosis of tonsillitis. Which clinical manifestation would likely have been noted in the child with this diagnosis? A. bark-like cough B. hoarseness C. erythema of the pharynx D. inability to make audible voice sounds

C. erythema of the pharynx Rationale: The child with tonsillitis may have a fever of 101°F (38.4°C) or higher, a sore throat, often with dysphagia (difficulty swallowing), hypertrophied tonsils, and erythema of the pharynx. The child with spasmodic laryngitis has a bark-like cough, hoarseness, and an inability to make audible voice sounds.

A client with bronchiectasis is admitted to the nursing unit. The primary focus of nursing care for this client includes A. teaching the family how to perform postural drainage. B. instructing the client on the signs of respiratory infection. C. implementing measures to clear pulmonary secretions. D. providing the client a low-calorie, high-fiber diet.

C. implementing measures to clear pulmonary secretions. Rationale: Nursing management focuses on alleviating symptoms and helping clients clear pulmonary secretions. Although teaching the family how to perform postural drainage and instructing the client on the signs of respiratory infection are important, they are not the nurse's primary focus. The presence of a large amount of mucus may decrease the client's appetite and result in inadequate dietary intake; therefore, the client's nutritional status is assessed and strategies are implemented to ensure an adequate diet.

The classification of Stage III of COPD is defined as A. at risk for COPD. B. mild COPD. C. severe COPD. D. very severe COPD. E. moderate COPD.

C. severe COPD. Rationale: Stage III is severe COPD. Stage 0 is at risk for COPD. Stage I is mild COPD. Stage II is moderate COPD. Stage IV is very severe COPD.

A nurse is caring for a client admitted with an exacerbation of asthma. The nurse knows the client's condition is worsening when he: A. sits in tripod position. B. has a pulse oximetry reading of 93%. C. uses the sternocleidomastoid muscles. D. wants the head of the bed raised to a 90-degree level.

C. uses the sternocleidomastoid muscles. Rationale: Use of accessory muscles indicates worsening breathing conditions. Assuming the tripod position, a 93% pulse oximetry reading, and a request for the nurse to raise the head of the bed don't indicate that the client's condition is worsening.

The nurse is caring for a client who has started therapy for tuberculosis. The client demonstrates an understanding of tuberculosis transmission when stating: A. "My tuberculosis isn't contagious after I take the medication for 24 hours." B. "I'm clear when my chest X-ray is negative." C. "I'm not contagious even if I have night sweats." D. "I'll follow airborne precautions until I have three negative sputum specimens."

D. "I'll follow airborne precautions until I have three negative sputum specimens." Rationale: A client is recommended to be maintained on airborne precautions until there are three consecutive acid-fast bacillus (AFB) sputum specimens that are negative. A client with nonresistant tuberculosis is no longer considered contagious when they show clinical evidence of decreased infection, such as significantly decreased coughing and fewer organisms on sputum smears. The medication may not produce negative acid-fast test results for several days. The client won't have a clear chest X-ray for several months after starting treatment. Night sweats are a sign of tuberculosis, but they don't indicate whether the client is contagious.

A client who has started therapy for drug-resistant tuberculosis demonstrates understanding of tuberculosis transmission when he says: A. "My tuberculosis isn't contagious after I take the medication for 24 hours." B. "I'm clear when my chest X-ray is negative." C. "I'm contagious as long as I have night sweats." D. "I'll stop being contagious when I have a negative acid-fast bacilli test."

D. "I'll stop being contagious when I have a negative acid-fast bacilli test." Rationale: A client with drug-resistant tuberculosis isn't contagious when he's had a negative acid-fast test. A client with nonresistant tuberculosis is no longer considered contagious when he shows clinical evidence of decreased infection, such as significantly decreased coughing and fewer organisms on sputum smears. The medication may not produce negative acid-fast test results for several days. The client won't have a clear chest X-ray for several months after starting treatment. Night sweats are a sign of tuberculosis, but they don't indicate whether the client is contagious.

The mother of a child with asthma tells the nurse that she occasionally gives her child the steroid medicine she takes for her rheumatoid arthritis when the child has a "flare-up" of asthma. "It's easier than going to the hospital or doctor every time a flare-up happens," the mother says. What is the best response by the nurse? A. "I understand that appointments can be annoying but steroid use can cause your child to have high blood sugar, peptic ulcers, slowed growth rate, and various other problems." B. "An adult should never give a child their medication. The doses may be very different." C. "As long as you only occasionally give your child the medication it shouldn't be a problem." D. "I'm sure it must be difficult to cope with the flare-ups, but there are many side effects from steroid use and the physician needs to monitor your child's asthma symptoms."

D. "I'm sure it must be difficult to cope with the flare-ups, but there are many side effects from steroid use and the physician needs to monitor your child's asthma symptoms." Rationale: Showing empathy for the parent is important when explaining the possibility of the steroid's side effects and the importance of the physician monitoring the child's asthma. Just listing all of the side effects of the steroid is not therapeutic communication and doesn't address the need for the child to be seen by the physician. Scolding the parent by telling her that she should never give her child her medication does not encourage good rapport. Giving the child the mother's medication even "occasionally" is not advisable.

The client asks the nurse to explain the reason for a chest tube insertion in treating a pneumothorax. Which is the best response by the nurse? A. "The tube will allow air to be restored to the lung." B. "The tube will drain secretions from the lung." C. "The tube will provide a route for medication instillation to the lung." D. "The tube will drain air from the space around the lung."

D. "The tube will drain air from the space around the lung." Rationale: Negative pressure must be maintained in the pleural cavity for the lungs to be inflated. An injury that allows air into the pleural space will result in a collapse of the lung. The chest tube can be used to drain fluid and blood from the pleural cavity and to instill medication, such as talc, to the cavity.

A client at risk for pneumonia has been ordered an influenza vaccine. Which statement from the nurse best explains the rationale for this vaccine? A. "Getting the flu can complicate pneumonia." B. "Influenza vaccine will prevent typical pneumonias." C. "Influenza is the major cause of death in the United States." D. "Viruses like influenza are the most common cause of pneumonia."

D. "Viruses like influenza are the most common cause of pneumonia." Rationale: Influenza type A is a common cause of pneumonia. Therefore, preventing influenza lowers the risk of pneumonia. Viral URIs can make the client more susceptible to secondary infections, but getting the flu is not a preventable action. Bacterial pneumonia is a typical pneumonia and cannot be prevented with a vaccine that is used to prevent a viral infection. Influenza is not the major cause of death in the United States. Combined influenza with pneumonia is the major cause of death in the United States.

The nurse is providing care to several children who have been brought to the clinic by the parents reporting cold-like symptoms. The nurse would most likely suspect sinusitis in which child? A. A 2-year-old with thin watery nasal discharge B. A 3-year-old with sneezing and coughing C. A 5-year-old with nasal congestion and sore throat D. A 7-year-old with halitosis and thick, yellow nasal discharge

D. A 7-year-old with halitosis and thick, yellow nasal discharge Rationale: The frontal sinuses, those most commonly associated with sinus infection, develop by age 6 to 8 years. Therefore, the 7-year-old would most likely experience sinusitis. In addition, this child also exhibits halitosis and a thick, yellow nasal discharge, other findings associated with sinusitis. Thin watery discharge in a 2-year-old is more likely to indicate allergic rhinitis. A 3-year-old with coughing and sneezing or a 5-year-old with nasal congestion and sore throat suggests the common cold.

What is an appropriate beverage for the nurse to give to a child who has a tonsillectomy earlier in the day? A. Chocolate ice cream B. Orange juice C. Fruit punch D. Apple juice

D. Apple juice Rationale: The child can have clear, cool liquids when fully awake.

A client's plan of care specifies postural drainage. Which action should the nurse perform when providing this noninvasive therapy? A. Administer the treatment with the client in a high Fowler or semi-Fowler position. B. Perform the procedure immediately following the client's meals. C. The client is instructed to avoid coughing during the therapy. D. Assist the client into a position that will allow gravity to move secretions.

D. Assist the client into a position that will allow gravity to move secretions. Rationale: In postural drainage, the client assumes a position that allows gravity to facilitate the draining of secretions from all areas of the lungs. Postural drainage is usually performed two to four times per day, before meals (to prevent nausea, vomiting, and aspiration) and at bedtime. Because the client usually sits in an upright position (i.e., high- or semi-Fowler position), secretions are likely to accumulate in the lower parts of the lungs. Several other positions are used in postural drainage so that the force of gravity helps move secretions from the smaller bronchial airways to the main bronchi and trachea. The client is encouraged to cough and remove secretions during postural drainage.

Which measure would be most effective in aiding bronchodilation in a child with laryngotracheobronchitis? A. Urging the child to continue to take oral fluids B. Administering an oral analgesic C. Teaching the child to take long, slow breaths D. Assisting with racemic epinephrine nebulizer therapy

D. Assisting with racemic epinephrine nebulizer therapy Rationale: Croup is a viral infection that causes inflammation and edema of the larynx, trachea, and bronchi. One form of treatment is the use of nebulized racemic epinephrine. Racemic epinephrine is an alpha adrenergic agent. It works on the mucosal vasoconstriction to reduce the edema. This increases the lumen of airways, allowing for better intake of air. A child in respiratory distress is unable to take slow, deep breaths. The child should not be offered fluids because this is an aspiration risk and analgesics will not reduce swelling.

A nurse is preparing to perform an admission assessment on a client with chronic obstructive pulmonary disease (COPD). It is most important for the nurse to review which of the following? A. Social work assessment B. Finances C. Chloride levels D. Available diagnostic tests

D. Available diagnostic tests Rationale: In addition to the client's history, the nurse reviews the results of available diagnostic tests. Social work assessment is not a priority for the majority of clients. Chloride levels are relevant to cystic fibrosis, not COPD. Immediate physiological status would be more important than finances.

Which statement is true about both lung transplant and bullectomy? A. Both procedures cure COPD. B. Both procedures treat end-stage emphysema. C. Both procedures treat patients with bullous emphysema. D. Both procedures improve the overall quality of life of a client with COPD.

D. Both procedures improve the overall quality of life of a client with COPD. Rationale: Treatments for COPD are aimed more at treating the symptoms and preventing complications, thereby improving the overall quality of life of a client with COPD. In fact, there is no cure for COPD. Lung transplant is aimed at treating end-stage emphysema and bullectomy is used to treat clients with bullous emphysema.

An 87-year-old client has been hospitalized with pneumonia. Which nursing action would be a priority in this client's plan of care? A. Nasogastric intubation B. Administration of probiotic supplements C. Bed rest D. Cautious hydration

D. Cautious hydration Rationale: Supportive treatment of pneumonia in the older adults includes hydration (with caution and with frequent assessment because of the risk of fluid overload in the older adults); supplemental oxygen therapy; and assistance with deep breathing, coughing, frequent position changes, and early ambulation. Mobility is not normally discouraged and an NG tube is not necessary in most cases. Probiotics may or may not be prescribed for the client.

A nurse should include what instruction for the client during postural drainage? A. Lie supine to rest the lungs. B. Sit upright to promote ventilation. C. Remain in each position for 30 to 45 minutes for best results. D. Change positions frequently and cough up secretions.

D. Change positions frequently and cough up secretions. Rationale: Clients who lie supine will have secretions accumulate in the posterior lung sections, whereas upright patients will pool secretions in their lower lobes. By changing positions, secretions can drain from the affected bronchioles into the bronchi and trachea and then be removed by coughing or suctioning.

The nurse is caring for a client following a thoracotomy. Which finding requires immediate intervention by the nurse? A. Heart rate, 112 bpm B. Moderate amounts of colorless sputum C. Pain of 5 on a 1-to-10 scale D. Chest tube drainage, 190 mL/hr

D. Chest tube drainage, 190 mL/hr Rationale: The nurse should monitor and document the amount and character of drainage every 2 hours. The nurse must notify the primary provider if drainage is ≥150 mL/hr. The other findings are normal following a thoracotomy and no intervention would be required.

A nurse is performing a focused assessment on a client with bronchiectasis. Which are the most prevalent signs and symptoms of this condition? Select all that apply. A. Radiating chest pain B. Wheezes on auscultation C. Increased anterior-posterior (AP) diameter D. Copious, purulent sputum E. Chronic cough

D. Copious, purulent sputum E. Chronic cough Rationale: Characteristic symptoms of bronchiectasis include clubbing of the fingers, chronic cough, and production of purulent sputum in copious amounts. Radiating chest pain, along with additional clinical indicators, are more indicative of a cardiovascular condition. Wheezes on auscultation are common in clients with asthma. An increased AP diameter is noted in clients with chronic obstructive pulmonary disease.

A nurse is assessing a client who comes to the clinic for care. Which findings in this client suggest bacterial pneumonia? A. Nonproductive cough and normal temperature B. Sore throat and abdominal pain C. Hemoptysis and dysuria D. Dyspnea and wheezing

D. Dyspnea and wheezing Rationale: In a client with bacterial pneumonia, retained secretions cause dyspnea, and respiratory tract inflammation causes wheezing. Bacterial pneumonia also produces a productive cough and fever, rather than a nonproductive cough and normal temperature. Sore throat occurs in pharyngitis, not bacterial pneumonia. Abdominal pain is characteristic of a GI disorder, unlike chest pain, which can reflect a respiratory infection such as pneumonia. Hemoptysis and dysuria aren't associated with pneumonia.

The nurse is assessing a patient who has been admitted with possible ARDS. Which finding would be evidence for a diagnosis of cardiogenic pulmonary edema rather than ARDS? A. Elevated white blood count B. Elevated troponin levels C. Elevated myoglobin levels D. Elevated B-type natriuretic peptide (BNP) levels

D. Elevated B-type natriuretic peptide (BNP) levels Rationale: Common diagnostic tests performed in patients with potential ARDS include plasma brain natriuretic peptide (BNP) levels, echocardiography, and pulmonary artery catheterization. The BNP level is helpful in distinguishing ARDS from cardiogenic pulmonary edema. Cardiogenic pulmonary edema is an acute event that results from heart failure, in which the cardiac chambers release atrial natriuretic peptide (ANP) and BNP to promote vasodilation and diuresis. BNP levels are not similarly elevated with ARDS.

The caregivers of a child report that their child had a cold and complained of a sore throat. When interviewed further they report that the child has a high fever, is very anxious, and is breathing by sitting up and leaning forward with the mouth open and the tongue out. The nurse recognizes these symptoms as those seen with which disorder? A. Spasmodic laryngitis B. Tonsillitis C. Laryngotracheobronchitis D. Epiglottitis

D. Epiglottitis Rationale: The child with epiglottitis may have had a mild upper respiratory infection before the development of a sore throat, and then became anxious and prefers to breathe by sitting up and leaning forward with the mouth open and the tongue out. The child with tonsillitis may have a fever, sore throat, difficulty swallowing, hypertrophied tonsils, and erythema of the soft palate. Exudate may be visible on the tonsils. The child with acute laryngotracheobronchitis develops hoarseness and a barking cough with a fever, cyanosis, heart failure; acute respiratory embarrassment can also result.

The nurse knows the mortality rate is high in lung cancer clients due to which factor? A. Increase in women smokers B. Increased incidence among the elderly C. Increased exposure to industrial pollutants D. Few early symptoms

D. Few early symptoms Rationale: Because lung cancer produces few early symptoms, its mortality rate is high. Lung cancer has increased in incidence due to an increase in the number of women smokers, a growing aging population, and exposure to pollutants but these are not directly related to the incidence of mortality rates.

What information should the nurse teach workers at a daycare center about RSV? A. RSV is transmitted through particles in the air B. RSV can live on skin or paper for up to a few seconds after contact C. RSV can survive on nonporous surfaces for about 60 minutes D. Frequent hand washing can decrease the spread of the virus

D. Frequent hand washing can decrease the spread of the virus Rationale: Meticulous hand washing can decrease the spread of organisms.

Which intervention is appropriate for the infant hospitalized with bronchiolitis? A. Position on the side with the neck slightly flexed B. Administer antibiotics as ordered C. Restrict oral and parenteral fluids if tachypneic D. Give cool, humidified oxygen

D. Give cool, humidified oxygen Rationale: Cool, humidified oxygen is given to relieve dyspnea, hypoxemia, and insensible fluid loss from tachypnea.

A client is admitted to a health care facility for treatment of chronic obstructive pulmonary disease. Which nursing diagnosis is most important for this client? A. Activity intolerance related to fatigue B. Anxiety related to actual threat to health status C. Risk for infection related to retained secretions D. Impaired gas exchange related to airflow obstruction

D. Impaired gas exchange related to airflow obstruction Rationale: A patent airway and an adequate breathing pattern are the top priority for any client, making Impaired gas exchange related to airflow obstruction the most important nursing diagnosis. Although Activity intolerance, Anxiety, and Risk for infection may also apply to this client, they aren't as important as Impaired gas exchange.

A client with chronic obstructive pulmonary disease has recently begun a new bronchodilator. Which therapeutic effect(s) should the nurse expect from this medication? Select all that apply. A. Negative sputum culture B. Increased viscosity of lung secretions C. Increased respiratory rate D. Increased expiratory flow rate E. Relief of dyspnea

D. Increased expiratory flow rate E. Relief of dyspnea Rationale: The relief of bronchospasm is confirmed by measuring improvement in expiratory flow rates and volumes (the force of expiration, how long it takes to exhale, and the amount of air exhaled) as well as by assessing the dyspnea and making sure that it has lessened. Increased respiratory rate and viscosity of secretions would suggest a worsening of the client's respiratory status. Bronchodilators would not have a direct result on the client's infectious process.

A nurse is caring for a client who was admitted with pneumonia, has a history of falls, and has skin lesions resulting from scratching. The priority nursing diagnosis for this client should be: A. Risk for falls. B. Ineffective breathing pattern. C. Impaired tissue integrity. D. Ineffective airway clearance.

D. Ineffective airway clearance. Rationale: Ineffective airway clearance is the priority nursing diagnosis for this client. Pneumonia involves excess secretions in the respiratory tract and inhibits air flow to the capillary bed. A client with pneumonia may not have an Ineffective breathing pattern, such as tachypnea, bradypnea, or Cheyne-Stokes respirations. Risk for falls and Impaired tissue integrity aren't priority diagnoses for this client.

A child is brought to the emergency department by his parents because he suddenly developed a barking cough. Further assessment leads the nurse to suspect that the child is experiencing croup. What would the nurse have most likely assessed? A. High fever B. Dysphagia C. Toxic appearance D. Inspiratory stridor

D. Inspiratory stridor Rationale: A child with croup typically develops a bark-like cough often at night. This may be accompanied by inspiratory stridor and suprasternal retractions. Temperature may be normal or slightly elevated. A high fever, dysphagia, and toxic appearance are associated with epiglottitis.

The nurse is assessing a client's potential for pulmonary emboli. What finding indicates possible deep vein thrombosis? A. Pain in the feet B. Coolness to lower extremities C. Decreased urinary output D. Localized calf tenderness

D. Localized calf tenderness Rationale: If the client were to complain of localized calf tenderness, the nurse would know this is a possible indication of a deep vein thrombosis. The area of tenderness could also be warm to touch. The client's urine output should not be impacted. Pain in the feet is not an indication of possible deep vein thrombosis.

A nurse is teaching the parents of a child diagnosed with cystic fibrosis about medication therapy. Which would the nurse instruct the parents to administer orally? A. Recombinant human DNase B. Bronchodilators C. Anti-inflammatory agents D. Pancreatic enzymes

D. Pancreatic enzymes Rationale: Pancreatic enzymes are administered orally to promote adequate digestion and absorption of nutrients. Recombinant human DNase, bronchodilators, and anti-inflammatory agents are typically administered by inhalation.

The nurse is discussing activity management with a client who is postoperative following thoracotomy. What instructions should the nurse give to the client regarding activity immediately following discharge? A. Walk 1 mile (1.6 km) 3 to 4 times a week. B. Use weights daily to increase arm strength. C. Walk on a treadmill 30 minutes daily. D. Perform shoulder exercises five times daily.

D. Perform shoulder exercises five times daily. Rationale: The nurse emphasizes the importance of progressively increased activity. The nurse also instructs the client on the importance of performing shoulder exercises five times daily. The client should ambulate with limits and realize that the return of strength will likely be gradual and likely will not include weight lifting or lengthy walks.

The nurse is caring for a child who has been admitted with a diagnosis of asthma. What laboratory/diagnostic tool would likely have been used for this child? A. Purified protein derivative test B. Sweat sodium chloride test C. Blood culture and sensitivity D. Pulmonary functions test

D. Pulmonary functions test Rationale: Pulmonary function tests are valuable diagnostic tools for the child with asthma and indicate the amount of obstruction in the bronchial airways, especially in the smallest airways of the lungs. Purified protein derivative tests are used to detect TB. Sweat sodium chloride tests are used for determining the diagnosis of cystic fibrosis. Blood culture and sensitivity is done to determine the causative agent as well as the anti-infective needed to treat an infection.

The nurse is examining a 5-year-old. Which sign or symptom is a reliable first indication of respiratory illness in children? A. Slow, irregular breathing B. A bluish tinge to the lips C. Increasing lethargy D. Rapid, shallow breathing

D. Rapid, shallow breathing Rationale: Tachypnea, or increased respiratory rate, is often the first sign of respiratory illness in infants and children. Slow, irregular breathing and increasing listlessness are signs that the child's condition is worsening. Cyanosis (a bluish tinge to the lips) or the degree of cyanosis present is not always an accurate indication of the severity of respiratory involvement.

The nurse is assessing the site of a client's Mantoux skin test. The client is HIV positive. The nurse notes the induration to be 10 mm. Which action will the nurse take next? A. Document the finding in the client's medical record B. Determine if the client has been exposed to tuberculosis. C. Place the client on droplet precaution. D. Schedule the client for a chest x-ray.

D. Schedule the client for a chest x-ray. Rationale: Diagnosis of tuberculosis (TB) is confirmed with a positive Mantoux test. An induration of 5 mm is considered positive for clients with HIV. If a client's Mantoux test is positive, the client is next scheduled for a chest x-ray to look for lung changes related to TB, or for a sputum smear test. These tests are done to determine if a client has latent or active TB, which will then determine the course of action for the client. The nurse will document the results; however, documentation is not priority. There is no need to ask about exposure since the test results indicate exposure. The client with active TB will be placed on airborne precautions.

The hospital case manager for a group of recently discharged clients with asthma is providing health education. Which aspect of client teaching would have the greatest impact on preventing readmissions? A. Alternative treatment modalities B. Family participation in care C. Pathophysiology of the disease process D. Self-care and the therapeutic regimen

D. Self-care and the therapeutic regimen Rationale: Knowledge about self-care and the therapeutic regimen would have the greatest impact on preventing admissions. For clients, the ability to understand the complex therapies of inhalers, anti-allergy and anti-reflux medications, and avoidance measures are essential for long-term control. Knowledge of alternative treatment modalities, such herbs, vitamins, or yoga, may help but is usually most effective as a complementary measure to an existing plan. Involving the family in care is important and can help the client with compliance, support, and encouragement, but ultimately the client is responsible for their own health. Understanding the pathophysiology of the disease process is important to include in education as it provides a better understanding in regards to causation and how it affects the body. However, how to physically manage asthma takes precedence over understanding in terms of readmission strategies.

A nurse is planning the care of a client with emphysema who will soon be discharged. What teaching should the nurse prioritize in the plan of care? A. Taking prophylactic antibiotics as prescribed B. Adhering to the treatment regimen in order to cure the disease C. Avoiding airplanes, buses, and other crowded public places D. Setting realistic short- and long-term goals

D. Setting realistic short- and long-term goals Rationale: A major area of teaching involves setting and accepting realistic short-term and long-term goals. Emphysema is not considered curable and antibiotics are not used on a preventative basis. The client does not normally need to avoid public places.

A child has had cold symptoms for more that 2 weeks, a headache, nasal congestion with purulent nasal drainage, facial tenderness, and a cough that increases during sleep. The nurse recognized that these symptoms are characteristic of which respiratory condition? A. Allergic rhinitis B. Bronchitis C. Asthma D. Sinusitis

D. Sinusitis Rationale: Sinusitis is characterized by signs and symptoms of a cold that do not improve after 14 days, a low-grade fever, nasal congestion and purulent nasal discharge, headache, tenderness, a feeling of fullness over the affected sinuses, halitosis, and a cough that increases when the child is lying down.

A client has just been diagnosed with small-cell lung cancer. The client asks the nurse why the doctor is not offering surgery as a treatment for the cancer. Which fact about lung cancer treatment should inform the nurse's response? A. The cells in small cell cancer of the lung are not large enough to visualize in surgery. B. Small cell lung cancer is self-limiting in many clients, and surgery should be delayed. C. Clients with small cell lung cancer are not normally stable enough to survive surgery. D. Small cell cancer of the lung grows rapidly and metastasizes early and extensively.

D. Small cell cancer of the lung grows rapidly and metastasizes early and extensively. Rationale: Surgery is primarily used for non-small cell lung cancer, because small cell cancer of the lung grows rapidly and metastasizes early and extensively. Difficult visualization and a client's medical instability are not the limiting factors. Lung cancer is not a self-limiting disease.

The nurse caring for a client recently diagnosed with lung disease encourages the client not to smoke. What is the primary rationale behind this nursing action? A. Smoking decreases the amount of mucus production. B. Smoke particles compete for binding sites on hemoglobin. C. Smoking causes atrophy of the alveoli. D. Smoking damages the ciliary cleansing mechanism.

D. Smoking damages the ciliary cleansing mechanism. Rationale: In addition to irritating the mucous cells of the bronchi and inhibiting the function of alveolar macrophage (scavenger) cells, smoking damages the ciliary cleansing mechanism of the respiratory tract. Smoking also increases the amount of mucus production and distends the alveoli in the lungs. It reduces the oxygen-carrying capacity of hemoglobin, but not by directly competing for binding sites.

The nurse is caring for a patient with pleurisy. What symptoms does the nurse recognize are significant for this patient's diagnosis? A. Dullness or flatness on percussion over areas of collected fluid B. Dyspnea and coughing C. Fever and chills D. Stabbing pain during respiratory movement

D. Stabbing pain during respiratory movement Rationale: When the inflamed pleural membranes rub together during respiration (intensified on inspiration), the result is severe, sharp, knifelike pain. The key characteristic of pleuritic pain is its relationship to respiratory movement. Taking a deep breath, coughing, or sneezing worsens the pain. Pleuritic pain is limited in distribution rather than diffuse; it usually occurs only on one side. The pain may become minimal or absent when the breath is held. It may be localized or radiate to the shoulder or abdomen. Later, as pleural fluid develops, the pain decreases.

The nurse assesses a patient for a possible pulmonary embolism. What frequent sign of pulmonary embolus does the nurse anticipate finding on assessment? A. Cough B. Hemoptysis C. Syncope D. Tachypnea

D. Tachypnea Rationale: Symptoms of PE depend on the size of the thrombus and the area of the pulmonary artery occluded by the thrombus; they may be nonspecific. Dyspnea is the most frequent symptom; the duration and intensity of the dyspnea depend on the extent of embolization. Chest pain is common and is usually sudden and pleuritic in origin. It may be substernal and may mimic angina pectoris or a myocardial infarction. Other symptoms include anxiety, fever, tachycardia, apprehension, cough, diaphoresis, hemoptysis, and syncope. The most frequent sign is tachypnea (very rapid respiratory rate).

The nurse is taking a health history for a 3-year-old girl suspected of having pneumonia who presents with a fever, chest pain, and cough. Which information places the child at risk for pneumonia? A. The child is a triplet. B. The child was a postmaturity date infant. C. The child has diabetes. D. The child attends daycare.

D. The child attends daycare. Rationale: Attending day care is a known risk factor for pneumonia. Being a triplet is a factor for bronchiolitis. Prematurity rather than postmaturity is a risk factor for pneumonia. Diabetes is a risk factor for influenza.

The nurse has assessed a 6-year-old child as having respiratory distress due to swelling of the epiglottis and surrounding structures. Which signs and symptoms would support this assessment? A. The child is pale and has vomited. B. The child has pale, elevated patches on the skin. C. The child is irritable and tachycardiac. D. The child is in tripod position.

D. The child is in tripod position. Rationale: Inflammation and swelling of the epiglottis and surrounding structures are common in children ages 2 to 7 years. The child will attempt to improve his/her airway by sitting forward and extending the neck forward with the jaw up, in a "sniffing position" (tripod position). Being pale, vomiting, and having elevated patches on the skin are not associated with epiglottis. Stridor, tachycardia, and the rapid onset are classical signs of epiglottitis.

The nurse is caring for a client who has been in a motor vehicle accident and is suspected of having developed pleurisy. Which assessment finding would best corroborate this diagnosis? A. The client is experiencing painless hemoptysis. B. The client's arterial blood gases (ABGs) are normal, but the client demonstrates increased work of breathing. C. The client's oxygen saturation level is below 88%, but the client denies shortness of breath. D. The client's pain intensifies when the client coughs or takes a deep breath.

D. The client's pain intensifies when the client coughs or takes a deep breath. Rationale: The key characteristic of pleuritic pain is its relationship to respiratory movement. Taking a deep breath, coughing, or sneezing worsens the pain. The client's ABGs would most likely be abnormal, and shortness of breath would be expected. Painless hemoptysis is not characteristic of pleurisy.

A nurse is providing health education to the family of a client with bronchiectasis. Which technique should the nurse prioritize teaching the client's family members? A. The correct technique for chest palpation and auscultation B. Techniques for assessing the client's fluid balance C. The technique for providing deep nasotracheal suctioning D. The correct technique for providing postural drainage

D. The correct technique for providing postural drainage Rationale: A focus of the care of bronchiectasis is helping clients clear pulmonary secretions; consequently, clients and families are taught to perform postural drainage. Chest palpation and auscultation and assessment of fluid balance are not prioritized over postural drainage. Nasotracheal suctioning is not normally necessary.

The nurse is taking a respiratory history of a newly admitted child. While documenting the symptoms the child has, what other item is important to document when taking a history on an altered respiratory status? A. The child's weight B. The child's diet C. The child's hospital history D. The triggers in the environment

D. The triggers in the environment Rationale: When assessing a respiratory history, it is very important for the nurse to find out what in the environment worsens the child's symptoms. These are called "triggers." The other choices would be part of a general health history.

The nurse caring for the child with asthma weighs the child daily. What is the most important reason for doing a daily weight on this child? A. To determine medication dosages B. To monitor the child's growth pattern C. To ensure that the child's food intake is adequate D. To determine fluid losses

D. To determine fluid losses Rationale: During an acute asthma attack the child may lose a great quantity of fluid through the respiratory tract and may have poor oral intake because of coughing and vomiting. Theophylline administration also has a diuretic effect, which compounds the problem. Weigh the child daily to help determine fluid losses. The child's weight is used to determine medication dosages, to ensure that the child is appropriately gaining weight and growing, and that the intake is adequate. However, the most important reason for a daily weight is to determine fluid loss.

A nurse is explaining to a client with asthma with a new prescription for prednisone what it is used for. What would be the most accurate explanation that the nurse could give? A. To ensure long-term prevention of asthma exacerbations B. To cure any systemic infection underlying asthma attacks C. To prevent recurrent pulmonary infections D. To gain prompt control of inadequately controlled, persistent asthma

D. To gain prompt control of inadequately controlled, persistent asthma Rationale: Prednisone is used for a short-term (3-10 days) "burst" to gain prompt control of inadequately controlled, persistent asthma. It is not used to treat infection or to prevent exacerbations in the long term.

Which vitamin supplements are necessary for children with CF? A. Vitamin C and calcium B. Vitamin B6 and B12 C. Magnesium D. Vitamins A, D, E, and K

D. Vitamins A, D, E, and K Rationale: Fat-soluble vitamins are poorly absorbed because of deficient pancreatic enzymes in children with CF; therefore supplements are necessary.

The nurse is caring for a child with a history of cystic fibrosis (CF). Which finding will the nurse report to the primary health care provider? A. Clubbing of the fingers B. Barrel chest C. Delayed puberty D. Wheezing

D. Wheezing Rationale: The nurse would report wheezing, as this indicates respiratory distress. Clubbing occurs with chronic respiratory illness. It is the result of increased capillary growth as the body attempts to supply more oxygen to distal body parts. Barrel chest refers to the shape the chest takes on in chronic respiratory illness. It takes the shape as chronically the lungs fill with air but are unable to fully expel the air. Delayed puberty is common in clients with cystic fibrosis and does not require reporting at this time.

The caregivers of an 8-year-old bring their child to the pediatrician and report that the child has not had breathing problems before, but since taking up lacrosse the child has been coughing and wheezing at the end of every practice and game. Their friend's child has often been hospitalized for asthma; they are concerned that their child has a similar illness. The nurse knows that because the problems seem to be directly related to exercise, it is likely that the child will be able to be treated with: A. decreased activity and increased fluids. B. corticosteroids and leukotriene inhibitors. C. removal of allergens in the home and school. D. a bronchodilator and mast cell stabilizers.

D. a bronchodilator and mast cell stabilizers. Rationale: Mast cell stabilizers are used to help decrease wheezing and exercise-induced asthma attacks. A bronchodilator often is given to open up the airways just before the mast cell stabilizer is used. Corticosteroids are anti-inflammatory drugs used to control severe or chronic cases of asthma. Leukotriene inhibitors are given by mouth along with other asthma medications for long-term control and prevention of mild, persistent asthma.

When preparing the room for an infant with bronchiolitis, which equipment is most important? A. a tracheostomy set B. a metered dose inhaler C. IV antibiotics D. oxygen tubing and facemask

D. oxygen tubing and facemask Rationale: Bronchiolitis is an acute inflammatory process in the bronchioles and small bronchi. The treatment is supportive oxygen therapy, suctioning, and hydration. Rarely is a tracheostomy set needed for care. An infant is not able to use a metered dose inhaler but nebulized bronchodilators may occasionally be needed. Bronchiolitis is most commonly associated with the respiratory syncytial virus (RSV), thus antibiotics would not be warranted in the treatment plan.

The nurse is assisting a child to the bed after tonsillectomy. How will the nurse place the child until fully awake? A. Fowler position B. semi-Fowler position C. supine position D. right lateral recumbent

D. right lateral recumbent Rationale: After a tonsillectomy and until fully awake, the nurse will place the child in a side-lying (right lateral recumbent) or face-down (prone) position to facilitate safe drainage of secretions. Once alert, the child may prefer to sit up (Fowler position) or have the head of the bed elevated (semi-Fowler position). If the child lays flat on the back (supine position), the child will need to be monitored for fluid drainage into the throat or lungs.

What is a symptom of allergic rhinitis (hay fever)? A. purulent secretions B. difficulty breathing C. laryngitis D. sinus pain

D. sinus pain Rationale: Rhinosinusitis is a bacterial infection of the paranasal sinuses. It can be an acute disorder or children may develop it as a chronic condition. The child will exhibit a cough, fever, halitosis (especially in preschoolers and older children), facial pain, eyelid edema, irritability, and poor appetite. Pharyngitis would occur if there was a throat infection, not a sinus infection. The drainage from the infected sinuses will generally be thick but it is not purulent. Respiratory difficulty is not seen because only the sinuses are involved.

The classification of Stage IV of COPD is defined as A. at risk for COPD. B. mild COPD. C. severe COPD. D. very severe COPD. E. moderate COPD.

D. very severe COPD. Rationale: Stage IV is very severe COPD. Stage 0 is at risk for COPD. Stage I is mild COPD. Stage II is moderate COPD. Stage III is severe COPD.

The nurse is assessing an infant who has been admitted with acute bronchiolitis. Which probable cause should the nurse suspect? A. bacterial infection B. environmental allergy C. prenatal complication D. viral infection

D. viral infection Rationale: Acute bronchiolitis is most commonly caused by a viral, not bacterial, infection. Neither allergies nor prenatal complications contribute to the development of this disorder.


Conjuntos de estudio relacionados

ISE Marketing Chapter 11: Managing Successful Products, Services, and Brands

View Set

AMSCO Chapter 24 + 25 MC Answers

View Set

Anatomy Exam 2: Practice Questions

View Set

Marketing 351 Chapter 12 Developing and Managing Products

View Set

CITI Training: Students Research

View Set

Physio SI: Chemistry of Physiology

View Set